TEST PAPER 2 Flashcards

1
Q

1.An intracardiac mass was incidentally detected during echocardiography in a patientundergoing preoperative assessment. Chest X-ray was non specific, showing an enlarged heart.Which one of the following statements regarding primary benign cardiac tumours is false?

A. Cardiac fibromas are homogenously low on T2W MRI.

B. Cardiac lipomas are high on T1W MRI.

C. Cardiac rhabdomyomas are associated with tuberous sclerosis.

D. Cardiac fibroelastomas usually arise from the pericardium.

E. Cardiac myxomas show heterogeneous signal on MRI.

A

1.D. Cardiac fibroelastomas usually arise from the pericardium.

Most lipomas appear to be subepicardial, expanding into the pericardial space. They typically have high signal on T1-weighted images and low signal on fat-suppressed images.
Fibroelastomas occur on cardiac valves, making them the most common neoplasm of the valves.They are hypointense mobile masses situated away from the free edge of the valves.

Fibroma is a neoplasm primarily of infants and children. Fibromas are homogeneouslyhypointense on T2-weighted images and show variable enhancement.

Rhabdomyomas are the most common benign cardiac tumour of childhood associated withtuberous sclerosis. They originate within the myocardium, typically in the ventricles, may be multiple, and are high on T2-weighted images.

The majority of myxomas manifest in adulthood between the fourth and seventh decades.
A minority will constitute part of an autosomal dominant syndrome known as Carney complex, characterised by myxomas, hyperpigmented skin lesions and extracardiac tumours such as pituitary adenomas, breast fibroadenomas and melanotic schwannomas. The classic triad of symptoms attributed to myxomas include cardiac obstructive symptoms related to obstruction to blood flow, embolic events and constitutional symptoms such as fever, malaise and weight loss. At MR imaging, the vast majority of myxomas demonstrate heterogeneous signal intensity.
The main differential diagnoses for myxoma include atrial thrombus or papillary fibroelastoma. Myxomas are more likely to arise anteriorly from the interatrial septum, whereas thrombus is more likely located posteriorly in the left atrium. In addition, myxomas enhance with gadolinium contrast material, whereas thrombi, in most cases, do not.

How well did you know this?
1
Not at all
2
3
4
5
Perfectly
2
Q

@# 2.A 40-year-old man undergoes a CT scan of the abdomen for recurrent abdominal pain.
The precontrast scan showed bilateral renal calculi. A post-contrast scan showed severalpancreatic lesions, measuring between 1 and 2 cm. What is the likely unifying diagnosis?

A. MEN I

B. MEN IIA

C. Insulinoma

D. Glucagonoma

E. NF1

A

2.A. MEN 1

Multiple endocrine neoplasia Type 1 is also known as Wermer syndrome. Inheritance isautosomal dominant with high penetrance. The male to-female ratio is 1:1. Organ involvement includes parathyroid hyperplasia (97%), pancreatic islet cell tumour (30% 80%), anterior pituitary gland tumour (15%—50%) and adrenocortical hyperplasia (33%-40%).

How well did you know this?
1
Not at all
2
3
4
5
Perfectly
3
Q

3.A 45-year old woman presents with a rapidly enlarging mildly painful breast mass over
a period of few months. An urgent ultrasound is performed. The ultrasound shows that themass measures 7 cm, filling up almost the entire breast with fluid-filled clefts in thetumour. What is the diagnosis?

A. Inflammatory carcinoma

B. Cystosarcoma phylloides

C. Complex breast cyst

D. Invasive lobular carcinoma

E. Breast lymphoma

A
  1. B. Cystosarcoma phyllodes

Phylloides tumour (PT) is a rare breast fibroepithelial neoplasm.

It is now generally accepted that PTs can be classified as benign, borderline or malignant.

Mammography and ultrasound are notorious for their inability to distinguish the benign or malignant histologic nature of PTs.

On US, they can be indistinguishable from fibroadenoma. They appear as an inhomogeneous, solid-appearing mass. A solid mass containing single or multiple, round or cleft-like cystic spaces and demonstrating posterior acoustic enhancement strongly suggests a diagnosis of PTs.Solid components of the tumour show vascularity on Doppler.

On MRI, well-defined margins with a round or lobulated shape and a septate inner structure have been described as characteristic morphologic signs. They are usually low on T1-weighted images and vary from low to very high signal on T2-weighted images. Some have described a silt-like pattern on MRIs of benign PTs; these appear as hyperintense slit-like fluid-filled spaces on T2-weighted images, with a low signal after enhancement. Solid areas of the tumour show enhancement with contrast.

How well did you know this?
1
Not at all
2
3
4
5
Perfectly
4
Q

4.Osteoid osteomas:

A. Are aggressive bone lesions with malignant potential

B. Are referred to as osteoblastomas when larger than 3 cm

C. Are typically cortical rather than subcortical based lesions

D. Typically require surgical curettage and resection

E. Are more common in women

A
  1. C. Arc typically cortical rather than subcortical-based lesions

Osteoid osteomas are benign and aggressive bone tumours that are more common in men and usually present clinically in patients less than 30 years of age.

They are most commonly based within the cortex, although they can also occur in any other area of the bone.

When larger than 2 cm, they are regarded as osteoblastomas.

Radiofrequency ablation is now a common and viable treatment for these lesions throughout the UK.

How well did you know this?
1
Not at all
2
3
4
5
Perfectly
5
Q

5.A 67-year-old known alcoholic man with acute onset neurological symptoms has beenreferred for an urgent CT brain to exclude haemorrhage. CT shows an isodense subdural onthe right, but the history’ of trauma was 24 hours earlier with no previous history of head
injury or fall. All of the following are causes of isodense subdural hacmatoma on CT following head trauma, except

A. A 2-week-old head injury with subdural haemorrhage

B. Acute subdural haematoma in a patient with Hb 7.5 g/dL

C. Chronic haematoma in a patient with coagulopathy

D. Patient with an associated arachnoid tear

E. Patient with leptomeningeal metastasis

A
  1. E. Patient with leptomeningeal metastasis

CT attenuation of blood in the subdural space remains denser than brain for 1 week and is less dense after 3 weeks. There is an interim period of 2 weeks when it is isodense to brain tissue.

In addition to variation in appearance over time, subdural haemorrhage may have a variable appearance in a setting of systemic disease like anaemia and coagulopathy.

An acute SDH can appear isodense in the following settings: anaemia with a haemoglobin concentration of <10 g/dL, admixture of CSF in the subdural space caused by an associated tear to the arachnoid layer and disseminated intravascular coagulation.

If there is chronic CSF leakage of venous blood, for example, if the patient has coagulopathy or is on anticoagulants/antiplatelet agents, chronic haematoma may look isodense rather than hypodense.

Meningeal metastasis may appear as hyperdensity on gvri and mimic subarachnoid haemorrhage

How well did you know this?
1
Not at all
2
3
4
5
Perfectly
6
Q
  1. A child presents with intermittent abdominal pain, vomiting, and a right upper-quadrant mass. On clinical examination, blood is noted on rectal examination. A clinical diagnosis of intussusception is made. Where is the most common site of intussusception in this population group?

A. Ileoileal

B. Ileocolic

C. Ileoileocolic

D. Colocolic

E. Jejunoileal

A
  1. B. Ileocolic

Intussusception is one of the most common causes of acute abdomen in infancy. Intussusception occurs when a portion of the digestive tract becomes telescoped into the adjacent bowel segment. This condition usually occurs in children between 6 months and 2 years of age. In this age group, intussusception is idiopathic in almost all cases. The vast majority of childhood cases of intussusception arc ileocolic.
US is highly accurate in the diagnosis of intussusception with a sensitivity of 98% 100% and a specificity of 88%-100%. US is also good at demonstrating alternative pathology. Hence, enema could be reserved for therapeutic purposes when US is available.

How well did you know this?
1
Not at all
2
3
4
5
Perfectly
7
Q
  1. A 29 year-old woman with fever, malaise, fatigue, intermittent pain and numbness in both hands and feet, and normal chest radiograph is referred for MRI thorax. MRI shows
    wall thickening of the origin of the right subclavian artery and both carotid arteries. What is the diagnosis?

A. Moyamoya disease

B. Takayasu arteritis

C. Churg-Strauss disease

D. PAN

E. Wegener’s granulomatosis

A
  1. B. Takayasu arteritis

Takayasu arteritis is a form of granulomatous vasculitis affecting large and medium-sized arteries, characterised by ocular disturbances and weak pulses in the upper extremities (pulseless disease).
It is associated with fibrous thickening of the aortic arch with narrowing of the origins of the great vessels at the arch. Takayasu arteritis can be limited to the descending thoracic and abdominal aorta. It is seen in young and middle-aged patients, especially Asian and women. The diagnosis is confirmed by a characteristic arteriographic pattern of irregular vessel walls, stenosis, post-stenotic dilatation, aneurysm formation, occlusion and evidence of increased collateral circulation.
Polyarteritis nodosa is a fibrinoid necrotising vasculitis that mainly involves small and mediumsized arteries of the muscles. Multiple aneurysm formation is a characteristic finding. The kidney is most commonly involved, followed by the GI tract, liver, spleen and pancreas. Positive ANCA titres (usually pANCA type) are found in variable percentages of patients.
Wegener s granulomatosis is a distinct clinicopathologic entity characterised by granulomatous vasculitis of the upper and lower respiratory tract together with glomerulonephritis.
Churg-Strauss syndrome is characterised by granulomatous vasculitis of multiple organ systems, particularly the lung, and involves both arteries and veins as well as pulmonary and systemic vessels.
Moyamoya disease is a progressive vasculopathy leading to stenosis of the main intracranial arteries. Characteristic angiographic features of the disease include stenosis or occlusion of the arteries of the circle of Willis, as well as the development of collateral vasculature that produces a typical angiographic image called clouds of smoke’ or ‘puff of cigarette smoke’.

How well did you know this?
1
Not at all
2
3
4
5
Perfectly
8
Q

@# 8. A slimly built 60-year-old woman presents with anorexia, diarrhoea, and weight loss. Barium meal shows multiple filling defects in the stomach with thickened gastric rugae. Colonoscopy shows multiple colonic polyps. The top differential is

A. Peutz-Jeghers syndrome

B. Familial adenomatous polyposis

C. Cronkhite-Canada syndrome

D. Cowden syndrome

E. Turcot’s syndrome

A
  1. C. Cronkhite-Canada syndrome

Cronkhite-Canada syndrome occurs in older patients with an average age of 60 with no familial predisposition.

The histologic appearance of the GI polyps resembles that of juvenile polyps, and they are characteristically distributed throughout the stomach.

They are commonly small, sessile and characterised by cystic dilatation of the glands and inflammation of the lamina propria.

Patients commonly present with abdominal pain, protein-losing diarrhoea, anorexia and weight loss. Dystrophic nail changes and alopecia usually appear after the onset of GI symptoms.

How well did you know this?
1
Not at all
2
3
4
5
Perfectly
9
Q
  1. A middle-aged woman presents to the breast clinic with a few weeks’ history of vague breast pain and lumpiness. On further direct questioning, she reveals a history of trauma to the breast a few months ago. Mammogram show’s a well-defined circular mass with central translucency and eggshell calcification. What is the most likely diagnosis?

A. Cystosarcoma phylloides

B. Complex breast cyst

C. Invasive lobular carcinoma

D. Fat necrosis

E. Fibroadenoma

A
  1. D. Fat necrosis

Fat necrosis is a frequently encountered cause of benign calcification, particularly when there is a history of trauma. It appears well circumscribed with translucent areas in the centre (homogenous fat density’ of the oil cyst). Occasionally, it shows curvilinear or eggshell calcification in the wall.

On US, it appears as a hypoechoic or anechoic mass with ill- or well-defined margins, with or without acoustic shadowing or as a complex cyst.

How well did you know this?
1
Not at all
2
3
4
5
Perfectly
10
Q
  1. A 60-year-old patient is admitted for progressive thoracic back pain with a low-grade fever, which has been going on for several weeks. Routine chest radiograph reveals no abnormality. An M RI of the spine reveals minimal intervertebral disc space reduction at T4/5 with hyperin tensity of the disc. There is adjacent vertebral end plate oedema and irregularity. A well-defined and
    predominantly high T2W signal collection is noted, extending beneath the anterior longitudinal ligament and cranially to reach the level of T2. What is the most likely diagnosis?

A. Pyogenic discitis

  1. Insufficiency fracture with ongoing collapse, complicated by chronic vertebral osteomyelitis

C. Degenerative disc disease with Modic Type II end plate changes

D. Pott disease

E. Vertebral lymphoma

A
  1. D. Pott disease

Infection usually begins in the anterior part of the vertebral body adjacent to the end plate.

Subsequent demineralisation of the end plate results in loss of definition of its dense margins on conventional radiographs.

These end plate changes allow the spread of infection to the adjacent intervertebral disk, resulting in a classic pattern of involvement of more than one vertebral body together with the intervening disks.

It also allows spread into the paraspinal tissues, resulting in the formation of a paravertebral abscess. However, if there is anterior subligamentous involvement of the spine, infection can extend both superiorly and inferiorly, with sparing of the intervertebral disks.

A normal chest radiograph is present in up to 50% of cases. In the later stages of disease, there is often vertebral collapse with a gibbus deformity.

Tuberculosis rarely affects the posterior vertebral elements (including the pedicles), in contrast to metastatic disease.

Anterior scalloping seen with subligamentous spread of infection can also be seen with paravertebral lymphadenopathy, secondary to metastases or lymphoma.

In differentiating tuberculosis from pyogenic infection, the clinical picture is as important as the radiologic features, with insidious onset of symptoms, a normal erythrocyte sedimentation rate, relevant respiratory symptoms and slow disease progression favouring a diagnosis of tuberculosis.

Radiologic features that favour this diagnosis include involvement of one or more segments; a delay in destruction of the intervertebral disks; a large, calcified paravertebral mass; and the absence of sclerosis.

Sarcoidosis can produce multifocal lesions of vertebrae and disks, along with paraspinal masses that appear identical to tuberculosis.

How well did you know this?
1
Not at all
2
3
4
5
Perfectly
11
Q
  1. A 16 year old boy with progressive extra-pyramidal symptoms, dementia and positive family history was sent for an MRI brain by his neurologist. MRI showed bilaterally symmetric hyperintense signal changes in the anterior medial globus pallidus with surrounding hypointensity in the globus pallidus on T2W images, commonly described as ‘eye of the tiger sign. Caudate was normal and no other areas of signal change was demonstrated. What is the diagnosis?

A. Wilson disease

B. Huntington disease

C. MELAS

I). Hallervorden-Spatz disease

E. CADASIL

A
  1. D. Hallervorden-Spatz disease

The ‘eye of the tiger sign represents marked low signal intensity of the globus palladi on T2-weighted MRI, surrounding a central, small hyperintense area. The sign is seen in what was once known as Hallervorden-Spatz (HS) syndrome but is now called neurodegeneration with brain iron accumulation (NBIA) or pantothenate kinase II (PANC2)-associated neurodegeneration.
The low signal is a result of excessive iron accumulation and the central high signal is attributed to gliosis, increased water content and neuronal loss with disintegration. Iron levels in blood and CSF are normal. HS is a neurodegenerative disorder associated with extrapyramidal dysfunction and dementia. The sign can also be seen in other extrapyramidal Parkinsonian disorders such as cortical-basal ganglionic degeneration, Steele-Richardson-Olszewski syndrome, and early onset levodopa-responsive Parkinsonism.

High signal in the basal ganglia, thalamus and midbrain is seen in Wilson disease.

Caudate is atrophic in Huntington disease.

CADASIL shows extensive white matter signal change

and MELAS shows multiple focal white matter signal changes.

How well did you know this?
1
Not at all
2
3
4
5
Perfectly
12
Q
  1. A 2-year-old boy presents with a large abdominal mass and there is suspicion of an underlying neuroblastoma. Which feature would be supportive of this diagnosis with the highest confidence?

A. Calcification is uncommon.

B. Well circumscribed.

C. Displaces major vessels rather than encasing them.

D. Encases major vessels but does not invade them.

E. Claw of renal tissue extends partially around the mass.

A
  1. D. Encases major vessels but does not invade them.

Neuroblastoma (NBL) is the most common extracranial tumour in childhood and commonly presents as an abdominal mass. Abdominal and pelvic tumours are usually large and heterogeneous, and approximately 80%-90% demonstrate calcification on CT scans. Low attenuation areas of necrosis or haemorrhage are frequently noted at CT. Vascular encasement and compression of the renal vessels, splenic vein, inferior vena cava, aorta, celiac artery and superior mesenteric artery may occur, and vascular invasion is rare. Regional invasion of the psoas and paraspinal musculature may occur, and invasion of the neural foramen into the epidural space is also frequent; these are better evaluated at MR imaging, as is regional organ invasion. Metastatic disease of the liver and lung are readily evaluated with CT. They are typically heterogeneous, variably enhancing and of relatively low signal intensity on T1 weighted images and high signal intensity on T2-weighted images.
Wilms tumour demonstrates a ‘daw’ of normal renal tissue around the tumour. In contrast to neuroblastoma, vessels are displaced rather than encased and vascular invasion occurs in approximately 5%-10% of cases.

How well did you know this?
1
Not at all
2
3
4
5
Perfectly
13
Q
  1. A 73-year-old woman with previous history of myocardial infarction was referred for a chest radiograph by her GP to exclude chest infection. No infective focus was identified but a focal bulge was noticed in the left heart border, with curvilinear calcification along the edge. What is the diagnosis?

A. Myocardial calcification

B. Right atrial calcification

C. Mitral annulus calcification

D. Calcified vegetations

E. Left ventricular aneurysm calcification

A
  1. E. Left ventricular aneurysm calcification

A cardiac false aneurysm is defined as a rupture of the myocardium that is contained by pericardial adhesion. It usually represents a rare complication of myocardial infarction, but it may also occur after cardiac surgery’, chest trauma and endocarditis. True left ventricular aneurysms are discrete, dyskinetic areas of the left ventricular wall with a broad neck. Unlike a true aneurysm, which contains some myocardial elements in its wall, the walls of a false aneurysm are composed of organised haematoma and pericardium only. Both demonstrate focal bulge to the cardiac contour and can calcify.
Marked delayed enhancement of the pericardium on dynamic enhanced MRI may help in differentiating a false aneurysm from a true one.

How well did you know this?
1
Not at all
2
3
4
5
Perfectly
14
Q

@# 14. A 70-year-old man presents with rectal bleeding. Flexible sigmoidoscopy shows a circumferential tumour in the upper third of the anal canal. An MRI performed for staging shows locoregional lymphadenopathy. The lymph node group most likely to be involved is

A. Superficial inguinal

B. Common iliac

C. Pudendal

D. External iliac

E. Paraortic

A
  1. A. Superficial inguinal

Metastatic spread to regional lymph nodes represents the most common mode of tumour spread from cancer of the anal canal and margin. Nodal metastasis is more likely in cases of larger tumour size or a poorly differentiated anal tumour. Metastasis most commonly occurs to the perirectal nodes, with inguinal nodal spread being the second most common location of nodal metastasis.

How well did you know this?
1
Not at all
2
3
4
5
Perfectly
15
Q
  1. A fit and healthy 25-year-old woman presents to the breast clinic with a small mobile non-tender breast lump that she noticed incidentally. An ultrasound is deemed as the first-line investigation; it reveals an extremely well-defined homogenous, hypoechoic oval mass with posterior acoustic shadowing. What is the most likely diagnosis?

A. Cystosarcoma phylloides

B. Fibroadenoma

C. Complex breast cyst

D. Invasive lobular carcinoma

E. Fat necrosis

A
  1. B. Fibroadenoma

Fibroadenomas are the most common cause of benign solid mass in the breast. On US, they appear round or oval, wider than tall, hypoechoic, well-defined and mostly homogenous and show a ‘hump and dip’ sign (small focal bulge to the contour with a contagious small sulcus), a thin echogenic pseudocapsule and rarely cither posterior acoustic enhancement (17%-25%) or posterior acoustic shadow (9%—11%).

How well did you know this?
1
Not at all
2
3
4
5
Perfectly
16
Q
  1. A 35-year-old weightlifter presents to the orthopaedic clinic with pain in the right shoulder. An initial radiograph is normal and no abnormality is identified on US. An MRI is suggested for further evaluation; it reveals increased T2W signal changes with fatty atrophy of the teres minor muscle. What is the likely diagnosis?

A. Parsonage Turner syndrome

B. Spinoglenoid notch paralabral cyst

C. Duchenne’s muscular dystrophy

D. Quadrilateral space syndrome

E. Acute rotator cuff tear

A
  1. D. Quadrilateral space syndrome

The anatomy of the suprascapular nerve renders it particularly susceptible to compression at the suprascapular notch and spinoglenoid notch.

The pattern of muscle denervation provides information about the duration of entrapment and can identify the site of neurologic compromise.

Acute denervation presents as hyperintensity of the supraspinatus and infraspinatus or of the infraspinatus muscle alone on fluid-sensitive sequences.

Chronic compression is shown as a reduction in muscle bulk and fatty infiltration of the involved muscles.

Involvement of both the supra- and infraspinatus muscles reflects proximal compression at the suprascapular notch,

whereas isolated infraspinatus denervation suggests compression at the spinoglenoid notch.

Quadrilateral space syndrome is a rare condition referring to an isolated compressive neuropathy of the axillary nerve. It generally results in isolated atrophy of the teres minor and, less commonly, of the deltoid, which appears as a reduction in muscle bulk and fatty infiltration with chronic compression.

Parsonage-Turner syndrome is an uncommon, self-limiting disorder characterised by sudden onset of non-traumatic shoulder pain associated with progressive weakness of the shoulder girdle musculature.

MR] is the technique of choice in patients with shoulder pain and weakness. It is sensitive for the detection of signal abnormalities in the shoulder girdle musculature related to denervation injury.

MRI is also useful in excluding intrinsic shoulder abnormalities that can produce symptoms similar to Parsonage-Turner syndrome such as rotator cuff tears, impingement syndrome and labral tears.

None of the findings or history would be compatible with an acute rotator cuff tear or Duchenne’s muscular dystrophy.

How well did you know this?
1
Not at all
2
3
4
5
Perfectly
17
Q
  1. A 45-year-old male patient with low back pain, rectal bleeding and faecal incontinence is investigated with CT and MRI. CT shows an enhancing soft-tissue mass replacing the sacrum with areas of amorphous calcifications. On MRI, the lesion shows low to intermediate signal on T1W and high signal on T2W images. What is your diagnosis?

A. Sacral meningocoele

B. Sacral chordoma

C. Central dural ectasia

D. Rhabdomyosarcoma

E. Sacrococcygeal teratoma

A
  1. B. Sacral chordoma

A chordoma is a tumour that derives from notochordal remnants. At imaging, it typically manifests as a large, destructive sacral mass with secondary soft-tissue extension. Radiographs may show sacral osteolysis with an associated soft-tissue mass and calcifications. CT shows bone destruction with an associated lobulated midline soft-tissue mass. Areas of low attenuation within the mass reflect the myxoid properties (high water content) of the tissue. Areas of punctate calcification often are noted.
At MR imaging, the most striking feature is the high signal intensity seen on T2-weighted images. High T2 signal intensity is a non-specific feature; however, the combination of high T2 signal intensity and a lobulated sacral mass that contains areas of haemorrhage and calcification is strongly suggestive of a chordoma. Chordomas tend to show hypointense or isointense signal relative to that in muscle on T1-weighted images, and contrast-enhanced images show a modest degree of heterogeneous enhancement in the soft-tissue components of the tumour. Areas of intrinsic Tl signal hyperintensity typically represent areas of haemorrhage or mucinous material. The tumour may cross the sacroiliac joint.

How well did you know this?
1
Not at all
2
3
4
5
Perfectly
18
Q
  1. A 3-year-old girl presents with a cough, temperature and hyperinflated left lower zone, elevating the left hilum. What is the likeliest diagnosis?

A. Congenital lobar emphysema

B. Viral pneumonia

C. Cystic fibrosis

D. Pulmonary sequestration

E. Inhaled foreign body

A
  1. E. Inhaled foreign body

Most inhaled foreign bodies are organic and may not be visible on chest X-ray. It is recognised that chest X-ray can be normal with inhaled foreign bodies and clinical suspicion needs to be high. The most common findings are unilateral, distal obstructive emphysema, followed by normal X-ray. Lower lobes are most commonly involved. Long-standing unrecognised foreign body may present with recurrent or non-resolving consolidation or unexplained segmental collapse.

How well did you know this?
1
Not at all
2
3
4
5
Perfectly
19
Q
  1. A 72-year-old man presents to the vascular surgeon with abdominal pain 4 months after endovascular repair of an abdominal aortic aneurysm. An emergency dual phase contrast-enhanced CT is performed. The unenhanced images reveal high-density material interposed between the stent and the wall of the aorta. There is further enhancement of this high-density area on arterial phase images. The graft and the attachments look intact.
    What is the most likely diagnosis?

A. Type I endoleak

B. Type II endoleak

C. Type III endoleak

D. Type IV endoleak

E. Type V endoleak

A
  1. B. Type II endoleak

In a Type I endoleak, there is poor apposition between one of the attachment sites of a stent graft and the native aortic or iliac artery wall, and blood leaks through this defect into the aneurysm sac. A Type I endoleak can be seen immediately after stent-graft deployment On CT, dense contrast collection is usually seen centrally within the sac and is often continuous with one of the attachment sites.
Type 11 endoleaks are the most common. They occur when there is retrograde flow of blood into the aneurysm sac via an excluded aortic branch, most commonly IMA or a lumbar artery. Many Type II endoleaks close spontaneously over time. CT shows peripheral or central location of acute haemorrhage.
Leakage of blood through the body of a stent graft results in a Type III endoleak. Type III endoleaks manifest as collections of haemorrhage or contrast material centrally within the aneurysm sac, usually distant from the attachment sites or native vessels.
Opacification of the aneurysm sac immediately after placement of a stent graft without a discernible source of leakage is designated a Type IV endoleak.
A Type V endoleak, or endotension, is characterised by continued growth of an excluded aneurysm sac without direct radiologic evidence of a leak.

How well did you know this?
1
Not at all
2
3
4
5
Perfectly
20
Q

@# 20. A 76-year old woman with 6 months’ history of progressive weight loss and altered bowel habits is referred for a CT scan of the abdomen and pelvis. The examination shows several hypoattenuating lesions on Segments II and III of the liver that are highly suspicious for malignancy. Blood biochemistry with tumour markers shows a normal AFP and CA 19-9 but raised CEA. LFTs are minimally deranged. The case is discussed at MDT.
What is the most likely differential diagnosis and further management plan out of the options given?

A. Primary hepatocellular carcinoma - liver biopsy

B. Hepatic adenoma - liver resection

C. Probable lung cancer - CT chest

D. Metastatic renal carcinoma - renal biopsy

E. Metastasis from colonic adenocarcinoma colonoscopy

A
  1. E. Metastasis from colonic adenocarcinoma - colonoscopy
How well did you know this?
1
Not at all
2
3
4
5
Perfectly
21
Q
  1. A late-middle-aged woman undergoes an MRI breast with contrast to investigate a clinically palpable nodularity. Fat-suppressed 3D images show a 3-cm well-defined abnormality with peripheral enhancement. Which of the following statements regarding enhancement pattern is incorrect?

A. Rim enhancement is a non-specific finding.

B. Fat content is a sign of benignity.

C. Homogenous enhancement is a sign of benignity.

D. Centripetal spread of contrast is more common in carcinomas.

E. Homogenous high T2 signal suggests malignancy.

A
  1. E.. Homogenous high T2 signal suggests malignancy.

Smooth margins, characterised by well-defined and sharply demarcated borders, are the feature with the highest benign lesion predictive value. Round and oval shapes have also been found to be predictive of benignity.

Homogeneous enhancement is highly suggestive of benign nature.

Fat content is specific to benign lesions, such as hamartoma, fibroadenoma, intramammary lymph nodes or fat necrosis.

In the case of fat necrosis, even if the lesion appears irregular in both shape and margin, with rim enhancement, the key to diagnosis is a fat internal signal on unenhanced sequences without fat suppression.

Strong hypersignal on non fat suppressed T2-weighted sequences is classically considered to be a clear sign of fibroadenoma, but it is non-specific and is seen in mucinous carcinomas, invasive ductal carcinomas, metaplastic carcinomas and intracystic papillary carcinomas (although it does not have a homogenously high T2 signal as fibroadenoma).

Although rim enhancement is regarded as suggestive of malignancy, a regular enhanced rim, which may be thick, may be seen around cysts, seroma (both of which are high T2 fluid signal) and fat necrosis (high T1 signal).

In relation to enhancement kinematics, a centrifugal contrast uptake pattern may help in diagnosing a fibroadenoma, whereas a centripetal spread of contrast is more common in carcinomas.

How well did you know this?
1
Not at all
2
3
4
5
Perfectly
22
Q

@# 22. Which statement is not associated with transient patellar dislocation?

A. The medial patellar retinaculum frequently demonstrates high T2W signal changes.

B. A tibial tuberosity to trochlear groove distance of <1.5 cm.

C. Trochlear dysplasia is a predisposing condition.

D. There is an increase in the ratio of the patellar tendon to the patellar length.

E. Bone contusions of the anterolateral aspect of the lateral femoral condyle.

A
  1. B. A tibial tuberosity to trochlear groove distance of <1.5 cm.

Transient patellar dislocation is the dislocation of the patella laterally and subsequent relocation.

Trochlear dysplasia, patella alta (increase in the ratio of the patella tendon to the patella length) and an increase in the tibial tuberosity trochlear groove (TT-TG) distance are associated factors.

TT-TG >20 mm is abnormal and 15-20 mm is considered borderline change.

TT-TG less than 15 mm is within normal limits.

Contusional marrow oedema is often seen in the medial patellar facet and the lateral femoral condyle.

The medial patellar retinaculum and/or medial patellofemoral ligament (MPFL) may be torn or show a pattern of strain injury.

How well did you know this?
1
Not at all
2
3
4
5
Perfectly
23
Q
  1. A 45-year-old patient with chronically progressive low back pain is referred for an MRI scan. Sagittal T1W and T2W MRI lumbar spine show dehydration of the L4/5 disc, with low signal to the disc on T2. High signal is noted in the adjacent end plates of the L4 and L5 vertebra on both the T1W and T2W sequences. There is no cortical destruction. How would you report the finding?

A. Modic Type I end plate change

B. Modic Type II end plate change

C. Andersson lesion

D. Modic Type III end plate change

E. Discitis

A
  1. B. Modic Type II end plate change

Modic described three types of reactive changes in the cancellous bone adjacent to the vertebral end plates.

Type I change is low on Tl and high on T2, representing oedema secondary to acute fibrovascular tissue invasion.

Type 2 change represents fatty replacement of red marrow, bright on Tl and T2. This leads to bony sclerosis, low on Tl and T2.

Occasionally the end plates become irregular and the degenerative process progresses to a destructive discovertebral lesion, simulating infective discitis. The key differentiation is signal of the disc, which is high in discitis and low in degeneration.

Andersson lesions refer to inflammatory involvement of the intervertebral discs by spondyloarthritis.

How well did you know this?
1
Not at all
2
3
4
5
Perfectly
24
Q
  1. A 4 year-old boy falls off his bike and complains of neck pain. Which of the following features is worrying for a serious injury on plain cervical X-rays?

A. Atlanto axial distance <5 mm

B. Displacement of 6 mm of the lateral masses relative to the dens

C. Absence of lordosis

D. Disruption of the spinolamellar line

E. Anterior subluxation of C2 on C3

A
  1. D. Disruption of the spinolamellar line

The atlantoaxial interval is defined as the distance between the anterior aspect of the dens and the posterior aspect of the anterior ring of the atlas. This distance should be 5 mm or less. Pseudospread of the atlas on the axis (‘pseudo-Jefferson fracture’) can be seen on anterior open-mouth radiographs. Up to 6 mm of displacement of the lateral masses relative to the dens is common in patients up to 4 years old and may be seen in patients up to 7 years old. On extension radiographs, overriding of the anterior arch of the atlas onto the odontoid process can be seen in 20% of healthy children.
In children, the C2-3 space and, to a lesser extent, the C3-4 space have a normal physiologic displacement. The absence of lordosis, although potentially pathologic in an adult, can be seen in children up to 16 years of age when the neck is in a neutral position.
In children, the flexion manoeuvre can increase the distance between the tips of the C1 and C2 spinous processes. Normal posterior intraspinous distance is a good indicator of ligamentous integrity and should not be more than 1.5 times greater than the intraspinous distance one level either above or below the level in question. Anterior wedging of up to 3 mm of the vertebral bodies should not be confused with compression fracture. Such wedging can be profound at the C3 level.
A prevertebral space of less than 6 mm at the level of C3 is considered normal in children.
In paediatric patients, widening of the prevertebral soft tissues can be a normal finding that is related to expiration.
Disruption of the spinolamellar line is a sign of injury.

How well did you know this?
1
Not at all
2
3
4
5
Perfectly
25
Q
  1. A 77-year-old man is brought into A&E with progressive increase in back pain. There is
    a history of known moderately large aortic aneurysm in the notes. A contrast CT is organised because the patient is haemodynamically stable. CT does not show any evidence of a ruptured aneurysm. All of the following are findings of pending rupture, except

A. Drape sign

B. Tangential calcium sign

C. Focal discontinuous intimal calcification

D. Retroperitoneal haematoma

E. High attenuation crescent

A
  1. D. Retroperitoneal haematoma

A retroperitoneal haematoma adjacent to an abdominal aortic aneurysm is the most common imaging finding of abdominal aortic aneurysm rupture.
The most common finding predictive of rupture is increase in aneurysm size, and thus it is the most common indicator for elective surgical management. Decreasing thrombus-lumen ratio is also predictive of increasing aneurysm size. The rest include drape sign, tangential calcium sign, focal discontinuous intimal calcification and high attenuation crescent.
High attenuation crescent represents an internal dissection of blood into either the peripheral thrombus or the aneurysm wall. It is one of the earliest and most specific imaging manifestations of the rupture process.

How well did you know this?
1
Not at all
2
3
4
5
Perfectly
26
Q
  1. A 56-year-old man with a history of PSC has an orthotopic liver transplant. He becomes unwell with a fever and acutely deranged LFTs on Day 4 post-operative. An ultrasound is subsequently performed. What is the likely cause of his symptoms?

A. Hepatic vein thrombosis

B. Portal vein thrombosis

C. Hepatic artery thrombosis

I). CBD ligation and cholangitis

E. Gangrenous cholecystitis with perforation

A
  1. C. Hepatic artery thrombosis

In the early post-operative period (<72 hours after transplantation), increased hepatic artery resistance (resistive index of >0.8) is a frequent finding, but resistance ordinarily returns to a normal level within a few days. Increased hepatic artery resistance is associated with older donor age and a prolonged period of ischaemia. Hie estimated incidence of hepatic artery thrombosis among liver transplant recipients is 4%-12% in adults and 42% in children. This is one of the most feared complications, as it may lead to fulminant hepatic necrosis. In addition, in liver grafts, biliary ducts are supplied exclusively by small branches of the hepatic artery; therefore, arterial thrombosis may lead to biliary ischaemia and necrosis. Prompt diagnosis of hepatic artery thrombosis is extremely important because early intervention (with thrombectomy, hepatic artery reconstruction or both) may allow graft salvage. However, most patients ultimately require retransplantation. Even after retransplantation, the mortality rate approaches 30%. Risk factors for hepatic artery thrombosis include a significant difference in hepatic artery calibre between the donor and the recipient, an interpositional conduit for the anastomosis, a previous stenotic lesion of the celiac axis, excessive duration of cold ischaemia time, ABO blood type incompatibility, cytomegalovirus infection and acute rejection.

How well did you know this?
1
Not at all
2
3
4
5
Perfectly
27
Q

@# 27. A young woman with a palpable nodule in the breast undergoes a contrast enhanced MRI breast for further evaluation. MRI demonstrates typical multiple non-enhancing internal septations. What is the diagnosis?

A. Fibrocystic change

B. Abscess

C. Atypia

D. Fibroadenoma

E. Hydatid cyst

A
  1. D. Fibroadenoma

Non-enhancing internal septations were initially described to have a high specificity in fibroadenoma diagnosis; however, this feature has recently been described in PTs and cancers and thus has little value when considered alone .Although morphological findings are important in lesion characterisation, breast cancers may have a benign appearance. In particular, 30% of familial breast cancers revealed a mass showing benign morphological features with a round or oval shape, smooth margins and homogeneous internal enhancement. It should be considered that all enhancing masses in women with genetic risks are suitable for biopsy when there is a lack of typical cyst or fat necrosis findings.

How well did you know this?
1
Not at all
2
3
4
5
Perfectly
28
Q
  1. A 21-year-old man presents with an acute knee injury following a violent tackle in
    a game of ice hockey. He is unable to weight bear. Plain film reveals a small elliptical bone fragment just adjacent to the lateral tibial plateau on the AP view. There is also an accompanying suprapatellar joint effusion. Based on the plain film findings and the mechanism of injury, what is the most commonly injured structure in this injury pattern?

A. Medial meniscus

B. Lateral meniscus

C. Anterior cruciate ligament

D. Posterior cruciate ligament

E. Lateral collateral ligament

A
  1. C. Anterior cruciate ligament

This is a classic plain film appearance of a Segond fracture. Segond fracture is an avulsion fracture at the proximal, non-articular aspect of the lateral tibia. The presence of this fracture is strongly associated with injury to other knee structures like the medial meniscus. However, it is the anterior cruciate ligament that is most commonly injured (75%-100%).

How well did you know this?
1
Not at all
2
3
4
5
Perfectly
29
Q
  1. A 36-year-old male patient with acute exacerbation of low back pain shows an 8 x 5 mm intermediate signal fragment lying in the epidural space with signal characteristics closely matching the lower lumbar discs. However, no definite continuity can be established with any of the local discs. Inflammatory markers and white cell count are normal. The most likely cause for this appearance would be

A. Disc extrusion

B. Disc protrusion

C. Meningioma

D. Schmorl node

E. Sequestrated disc

A
  1. E. Sequestrated disc

Protrusion is present if the greatest distance between the edges of the disc material presenting outside the disc space is less than the distance between the edges of the base of that disc (wider than tall). Extrusion is present when, in at least one plane, any one distance between the edges of the disc material beyond the disc space is greater than the distance between the edges of the base of the disc material or when no continuity exists between the disc material beyond the disc space and that within the disc space (taller than wide). The latter form of extrusion is best specified as sequestration if the displaced disc material has lost continuity completely with the parent disc. The term migration may be used to signify displacement of disc material away from the site of extrusion.

Herniated discs in the craniocaudal (vertical) direction through a gap in the vertebral end plate are referred to as intravertebral herniations (Schmorl nodes).

How well did you know this?
1
Not at all
2
3
4
5
Perfectly
30
Q
  1. A young boy undergoes an MCUG, which shows reflux of contrast into the right ureter and pelvicalyceal system. The ureter and pelvicalyceal system are not dilated. What grade is the reflux?

A. 1

B. 2

C. 3

D. 4

E. 5

A
  1. B. 2

Grading of reflux:
Grade I: Reflux into distal ureters
Grade II: Reflux into collecting system (without calyceal dilatation/blunting)
Grade III: All of the above, plus mild dilatation of the pelvis and calices
Grade IV: All of the above, plus moderate dilatation (clubbing of calices)
Grade V: All of the above, plus severe tortuosity of the ureter
Prognosis: Grades I—III VUR resolve with maturation of the ureterovesical junction; Grades IV and V require surgery to avoid renal scarring, renal impairment and/or hypertension.

How well did you know this?
1
Not at all
2
3
4
5
Perfectly
31
Q

@# 31. A 66-year-old man with central chest pain radiating to the back is brought into the A&E department. A chest X-ray is read as unremarkable and a contrast CT is organised. The contrast CT shows an acute dissection flap in the aortic arch at the origin of the left common
carotid trunk extending through the descending thoracic aorta, into the proximal
abdominal aorta at the level of the renal arteries. Which of the following best classifies this dissection type?

A. DeBakey 1 - Stanford A

B. DeBakey 2 - Stanford A

C. DeBakey 3 - Stanford A

D. DeBakey 1 - Stanford B

E. DeBakey 2 - Stanford B

A
  1. A. DeBakey 1 - Stanford A
How well did you know this?
1
Not at all
2
3
4
5
Perfectly
32
Q
  1. A 22-year-old man with mucocutaneous skin pigmentation around the lips and hand presents to the A&E department with acute colicky abdominal pain. Plain radiograph and subsequent CT of the abdomen show multiple dilated loops of small bowel with a transition point in the distal ileum associated with an ileo-ileal intussusception. What is the most likely diagnosis?

A. Gardner’s syndrome

B. Peutz-Jeghers syndrome

C. Cowdens syndrome

D. Turcot syndrome

E. TAR syndrome

A
  1. B. Peutz-Jeghers syndrome

Peutz-Jeghers syndrome is a disorder characterised by mucocutaneous pigmentation and gastrointestinal hamartomas. The syndrome is an autosomal, dominant, inherited trait and occurs with equal frequency in male and female subjects. Mucocutaneous pigmentation, a characteristic feature of this syndrome, is manifested by melanotic deposits around the nose, lips and buccal mucosa hands and feet.

Peutz-Jeghers polyps can be found from the stomach to the rectum and are most common in the small intestine, particularly the jejunum and ileum.``

How well did you know this?
1
Not at all
2
3
4
5
Perfectly
33
Q

@# 33. All of the following are accepted indications for MRI breast in breast cancer imaging, except

A. Response to chemotherapy

B. Patients with breast augmentation

C. High-risk screening

D. Differentiate residual disease from post-surgical scar

F. . Breast cancer recurrence

A
  1. D. Differentiate residual disease from post-surgical scar

MRI of the breast has evolved into an important adjunctive tool with multiple indications in breast imaging, as recommended by US and European guidelines. Breast MRI is currently the most sensitive detection technique for breast cancer diagnosis. The indications are staging before treatment planning, screening of high-risk women, evaluation of response to neoadjuvant chemotherapy, patients with breast augmentation or reconstruction, occult primary breast cancer, breast cancer recurrence, identifying residual tumour in positive surgical margins and characterisation of equivocal findings. Differentiating early post-operative scarring from residual breast tumour is not possible because of similar enhancement characteristics of post-surgical scarring.

How well did you know this?
1
Not at all
2
3
4
5
Perfectly
34
Q
  1. A 66-year-old woman presents with back pain that radiates down the left anterior thigh towards the medial aspect of the knee. What are the MRI lumbar spine findings that you may expect based on the clinical history?

A. L1/2 generalised disc bulge with left lateral recess stenosis

B. L1/2 disc bulge with left foraminal stenosis

C. L2/3 generalised disc bulge with left lateral recess stenosis

D. L2/3 generalised disc bulge with left neural foraminal stenosis

E. L3/4 generalised disc bulge with left lateral recess stenosis

A
  1. C. L2/3 generalised disc bulge with left lateral recess stenosis

Medial aspect of the knee corresponds to the L3 dermatome. lateral recess stenosis at L2/3 will affect the transiting L3 nerve root, whereas foramina! stenosis will affect the exiting L2 nerve root.

How well did you know this?
1
Not at all
2
3
4
5
Perfectly
35
Q
  1. A CT abdomen done on a 38-year-old woman with pancreatitis incidentally shows the presence of split cord syndrome with a central bony bridge in the vertebra. A horseshoe kidney is evident. All of the following are other well-described associations seen in patients with diastematomyelia, except

A. Thickened filum

B. Club foot

C. Cord tethering

D. Chiari malformation

E. Congenital dislocation of hips

A
  1. E. Congenital dislocation of hips

Split cord malformations (SCMs) are relatively rare forms of occult spinal dysraphism and tethered spinal cord syndrome. SCMs are of two types.

Type I consists of two hemicords, each contained within its own dural sheath and separated by a median bony spur,

and Type II consists of two hemicords housed in a single dural tube separated by a fibrous median septum.

There are recognised associations with myelomeningocele, Chiari malformation, tethering of cord, hypertrichosis, nevus, lipoma, dimple or haemangioma overlying the spine, clubfoot (50%), muscle wasting and progressive scoliosis.

How well did you know this?
1
Not at all
2
3
4
5
Perfectly
36
Q
  1. Which of the following is false?
A
  1. B. Normal trachea and anterior oesophageal indentation Aberrant right subclavian artery

Aberrant right subclavian artery with a left-sided aortic arch or aberrant left-sided subclavian artery with a right-sided aortic arch both result in posterior impression on the oesophagus on a barium swallow, with normal appearance of the trachea. An aberrant left pulmonary artery or pulmonary vascular sling runs in between the trachea and oesophagus, resulting in an anterior indentation on the oesophagus and a posterior indentation on the trachea. Other entities that can result in anterior indentation on the oesophagus arc a bronchogenic cyst, trachea-oesophageal node or a tracheal neoplasm extending posteriorly.
Anterior tracheal, posterior tracheal and lateral oesophageal impression occurs with double aortic arch. The right arch is higher than the left, resulting in an S-shaped oesophagogram on AP view. Reverse ‘3’ indentation of the oesophagus and normal trachea occurs with coarctation of the aorta.

How well did you know this?
1
Not at all
2
3
4
5
Perfectly
37
Q

@# 37. An 8-week old boy presents with profound cyanosis with associated congestive cardiac failure. Imaging demonstrates a localised concurrent aortic coarctation. The likeliest underlying diagnosis would be

A. Tetralogy of Fallot

B. Truncus arteriosus

C. Transposition of the great arteries

D. Hypoplastic left heart syndrome

E. Tricuspid atresia

A
  1. D. Hypoplastic left heart syndrome

Hypoplastic left heart syndrome presents with early onset (days) of cyanosis and heart failure, leading to collapse and death in a few weeks of life. Associated cardiac malformations include pre- and post-ductal coarctation of the aorta, PDA, VSD, patent foramen ovale and so on.
Truncus arteriosus presents with minimal cyanosis in newborn infants; signs of heart failure are usually absent. Heart failure is evident in older infants.
Tetralogy of Fallot presents in early infancy with cyanosis, usually not present in early infancy, leading to clubbing; dyspnoea, heart failure, failure to thrive and paroxysmal hypercyanotic spells. X-ray shows a boot-shaped heart with oligaemic lungs.
Transposition of the great arteries is a medical emergency. Infants usually present in the first few hours or days with worsening duct-dependent cyanosis. Hypoxia is severe, but heart failure is not a feature. X-ray shows an ‘egg on end’ or ‘egg on string’ appearance.
Tricuspid atresia presents in the first few days of life with increasing cyanosis; other clinical features are dependent on associated PDA or VSD.

How well did you know this?
1
Not at all
2
3
4
5
Perfectly
38
Q

@# 38. All the following are useful features for differentiating true lumen from false lumen, except

A. In case of lumen wrapping, the inner lumen is true.

B. Beak sign indicates a false lumen.

C. A false lumen is often larger than a true lumen.

D. A cobweb sign demarcates the true lumen.

E. Intimal calcification surrounds the true lumen.

A
  1. D. A cobweb sign demarcates the true lumen.

The beak sign and a larger cross-sectional area were the most useful indicators of the false lumen for both acute and chronic dissections. Features generally indicative of the true lumen included outer wall calcification and eccentric flap calcification. In cases showing one lumen wrapping around the other lumen in the aortic arch, the inner lumen was invariably the true lumen. Outer wall calcification always indicated the true lumen on scans of acute dissections. False lumen thrombus was significantly more frequent in chronic dissections than acute dissections. Cobwebs are specific for the false lumen but are only rarely observed.

How well did you know this?
1
Not at all
2
3
4
5
Perfectly
39
Q
  1. Contrast enhanced CT performed on a 52 year old man showed a hypodense liver with low attenuation rim around the portal tracts. The following differential diagnoses are recognised causes of periportal halo sigh, except

A. Blunt trauma to the liver

B. Hepatitis

C. Congestive heart disease

D. Obesity

E. Cardiac tamponade

A
  1. D. Obesity

Periportal halos are defined as circumferential zones of decreased attenuation identified around the peripheral or subsegmental portal venous branches on contrast-enhanced CT. These halos probably represent fluid or dilated lymphatics in the loose areolar zone around the portal triad
structures. While this CT finding is non-specific, it is abnormal and should prompt close scrutiny of the liver in search of an underlying aetiology.
Periportal halos, which may be due to blood, are commonly seen in patients with liver trauma. Periportal oedema may cause this sign in patients with congestive heart failure and secondary liver congestion, hepatitis, or enlarged lymph nodes and tumours in the porta hepatis, which obstruct lymphatic drainage. This CT sign has also been observed in liver transplants (probably secondary to disruption and engorgement of lymphatic channels) and in recipients of bone marrow transplants, who might develop liver oedema from microvenous occlusive disease. Although the precise pathophysiologic basis of periportal tracking has not been proven, it represents a potentially important CT sign of occult liver disease.

How well did you know this?
1
Not at all
2
3
4
5
Perfectly
40
Q
  1. All of the following are technical requirements for performing an MRI breast, except

A. Supine position

  1. Breast coil

C. Intravenous contrast

D. Thin slices

E. T1W images

A
  1. A. Supine position

There are several prerequisites for maximising the sensitivity and specificity of breast MRI, including the following:
* High magnetic field strength with a highly homogeneous magnetic field.
* Bilateral image acquisition with a prone-positioning bilateral breast coil.
* Unenhanced imaging with a T2-weighted and 3D T1-weighted pre- and post-IV gadolinium.
* Selection of a phase-encoding direction minimises artefacts.
* Homogeneous fat suppression.
* Thin-section acquisitions (section thickness of 3 mm or less).
* Pixel size of less than 1 mm in each in-plane direction.
* Temporal resolution of less than 2 minutes for imaging of both breasts.

How well did you know this?
1
Not at all
2
3
4
5
Perfectly
41
Q

@# 41. A 74 year-old man presents with neck pain, with right upper-arm pain and radicular symptoms at the lateral aspect of the forearm and tingling in the thumb. What is the most likely finding on the MRI?

A. Central disc bulge at C3/4 with severe cord compression

B. Right foraminal disc osteophyte at C2/3

C. Right foraminal disc osteophyte at C4/5

D. Right foraminal disc osteophyte at C5/6

E. Right foraminal disc osteophyte at C3/4

A
  1. D. Right foraminal disc osteophyte at C5/6

Lateral aspect of the forearm and the thumb corresponds to the C6 dermatome. Foraminal osteophyte at C5/6 will impinge upon the exiting C6 nerve root. (cf. foraminal osteophyte at a thoracic or lumbar level, e.g., T4/5 or L4/5, which will impinge upon the exiting T4 or L4 nerve roots, subject to the discrepancy between number of cervical vertebra and cervical roots. Note that the exiting root at C7/T1 is C8.)

How well did you know this?
1
Not at all
2
3
4
5
Perfectly
42
Q

@# 42. A 35-year-old man involved in a major RTA undergoes a lateral view of the cervical spine m the resus on arrival. All of the following are features associated with atlanto occipital dislocation, except

A. Soft-tissue swelling anterior to C2 by >10 mm.

B. Basion dens interval >12 mm.

C. Odd’s ratio >1.

D. X-ray can often be normal.

E. Incongruity of articular surface of atlas and occipital condyles.

A
  1. D. X-ray can often be normal.

Atlanto-occipital dislocation shows the following on lateral radiograph of the cervical spine: >10 mm soft-tissue swelling anterior to C2, with pathological convexity (80%), basion-dens interval of >12 mm, odd’s ratio (distance between the basion and the posterior arch of the atlas divided by opisthion and anterior arch of atlas) >1, and basion-posterior axial line interval >12 mm anterior/>4 mm posterior to axial line.
Direct signs include loss of congruity of articular surfaces of atlas and occipital condyle. Normal X-ray in the presence of atlanto-occipital dislocation is rare.

How well did you know this?
1
Not at all
2
3
4
5
Perfectly
43
Q

@# 43. A child presents with an abnormally shaped cranium. There is hypertelorism and the skull vault appears short and widened. The anterior cranial fossa appears shortened. Which sutures are likely to have fused early?

A. Sagittal

B. Coronal

C. Metopic

D. Lambdoid

E. Unilateral coronal and lambdoid

A
  1. B. Coronal

The appearance here describes brachycephaly. Craniosynostosis is the premature fusion of cranial sutures and may be isolated or may present as part of a craniofacial syndrome. It typically alters the shape of the cranial vault. Broad categories include simple craniosynostosis, involving only one suture, or compound craniosynostosis, where two or more sutures are involved.

How well did you know this?
1
Not at all
2
3
4
5
Perfectly
44
Q

@# 44. A 37 year old woman with newly diagnosed hypertension undergoes an MRA, which shows alternate areas of stricture and dilatation to the right renal artery, sparing the origin. Which of the following is the least likely association?

A. String-of-beads appearance of the contralateral renal artery

B. String-of-beads appearance of the extracranial internal carotid artery

C. String of-beads appearance of the retinal artery

D. String-of-beads appearance of the vertebral artery

E. String-of-beads appearance of the SMA

A
  1. C. String-of-beads appearance of the retinal artery

Fibromuscular dysplasia (FMD) is an idiopathic, segmentary, non-inflammatory and non atherosclerotic disease that can affect all layers of both small- and medium-calibre arteries. Vascular loops, fusiform vascular ectasia and a string-of-beads aspect are typical presentations. Arterial dissection, aneurysm and subarachnoid haemorrhage are less typical radiologic presentations.
The affected arteries are mainly the renal arteries, extracranial carotid and vertebral arteries, mesenteric arteries and intracranial carotid arteries. Central retinal artery occlusion is a rarely recognised feature of fibromuscular dysplasia.

How well did you know this?
1
Not at all
2
3
4
5
Perfectly
45
Q
  1. A 40 year-old man with a known history of primary haemochromatosis was referred for assessment of the liver with an MRI scan. Which option shows the expected signal characteristics of the liver with iron deposition?

A. Hyperintense liver relative to spleen on T1 and T2

B. Hypointense liver relative to spleen on T1 and T2

C. Isointense liver relative to spleen on T1 and T2

D. Signal drop out in the liver on out-of-phase Tl imaging

E. High-signal liver on the high B-value diffusion-weighted sequences

A
  1. B. Hypointense liver relative to spleen on T1 and T2

MR imaging is the best non invasive method for measuring the level of iron in the liver for the purposes of confirming the diagnosis, determining the severity, and monitoring therapy with high sensitivity, specificity, and positive and negative predictive values. The accumulation of iron ions in
the tissues, because of the super-paramagnetic properties of the ions, causes local distortion in the magnetic fields and relaxation of the spins, which results in shortening of the longitudinal relaxation time (T1), the transverse relaxation time (T2) and particularly the transverse relaxation time as affected by magnetic field inhomogeneity (T2). This effect causes a loss of signal intensity in the affected organs that is proportional to the iron deposition. In the general protocol applied to an abdominal study, it is not possible to estimate the hepatic iron concentration, although most of the time it is possible to diagnose iron overload. This can be done by using ‘dual sequence’ (gradient in and out of phase) MR imaging, which demonstrates decreased signal intensity in the affected tissues on the in-phase images compared with the out of-phase images. That effect is the opposite of the effect observed in patients with steatosis. This occurs because the echo time of the in-phase sequence is usually higher than that of the out-of-phase sequence; therefore, the in-phase pulse sequence is more sensitive to iron deposits because of the increased T2 effect.

How well did you know this?
1
Not at all
2
3
4
5
Perfectly
46
Q
  1. A 47-year-old woman presents to the breast clinic with a palpable lump. Mammography shows a well-circumscribed round mass with mixed dense and radiolucent areas surrounded by a thin radiopaque capsule. Targeted ultrasound performed m the clinic shows a sharply defined, encapsulated, round, heterogeneous mass with echo texture similar to surrounding breast MRI breast reveals a lesion heterogeneous on T1W and T2W images. What findings on MRI confirm the diagnosis?

A. Non-enhancing internal septations

B. Slowly enhancing peripheral rim

C. Solid homogenous enhancing lesion with w’ell-defined margins

D. Peripheral hyperintense cystic spaces on T2W images

E. Thin hypointense pseudocapsule and fat content on T1W images

A
  1. E. Thin hypointense pseudocapsule and fat content on T1W images

Fat necrosis appears as well circumscribed with translucent areas in the centre (homogenous fat density of the oil cyst). Occasionally, it shows curvilinear or eggshell calcification in the wall.
On US, it appears as a hypoechoic or anechoic mass with ill- or well-defined margins, with or without acoustic shadowing or as a complex cyst. On MRI, even if the lesion appears irregular in both shape and margin, with a rim enhancement, the key to diagnosis is a fat internal signal on unenhanced sequences without fat suppression (high on T1-weighted images).

How well did you know this?
1
Not at all
2
3
4
5
Perfectly
47
Q
  1. What is the primary role of a T2W fluid-sensitive sequence in an MRI shoulder arthrographic study with intra-articular administration of gadolinium?

A. To check for glenoid labral tears

B. To check for articular cartilage integrity

C. To check for extra-articular fluid

D. To determine whether there is a full-thickness rotator cuff tear with communication to the bursae

E. To increase the visual contrast of the soft-tissue structures

A
  1. C. To assess for extra-articular fluid

The purpose of the T2-weighted fluid-sensitive sequence in MR shoulder arthrography is the identification of extra-articular fluid, for example, bursal fluid or paralabral cysts, which may not be visualised on the isolated Tl weighted post-contrast sequences, particularly if there is no intra- articular communication.
T2-weighted imaging also helps to identify abnormal fluid in the rotator cuff tendons by eliminating magic angle because of high TE.

How well did you know this?
1
Not at all
2
3
4
5
Perfectly
48
Q
  1. A 35-year-old man involved in a major RTA undergoes a facial series due to marked swelling and bruising of the face. All of the following are true of Lefort fractures, except

A. By definition, the pterygoid plates have to be fractured.

B. Lefort 1 involves the maxilla and medial wall of the orbit.

C. Lefort III involves the maxilla and the medial and lateral wall of the orbit

D. Lefort III is also called craniofacial disjunction.

E. Lefort II fracture is also called pyramidal fracture

A
  1. B. Lefort I involves the maxilla and medial wall of the orbit.

Lefort fractures always involve the pterygoid process/plates.

Lefort I fractures are transverse maxillary fractures, involving the alveolar ridge and inferior wall of maxillary sinus. There is detachment of the teeth bearing alveolar process of maxilla. The orbits are not involved.

Lefort II fractures are also called pyramidal fractures and they involve the maxilla and the medial wall of the orbit. They can be unilateral.

Lefort III fractures are also called craniofacial disassociation or disjunction and involves the maxilla and medial and lateral wall of the orbit (zygomatic arch).

How well did you know this?
1
Not at all
2
3
4
5
Perfectly
49
Q
  1. A 46-year-old woman with Marfan’s syndrome presents to the A&E department with progressive worsening of central chest pain radiating to the back. Clinically, there is a differential blood pressure between the two arms, and aortic dissection is strongly suspected. CECT chest confirms the dissection and shows its distribution involving the arch of the aorta and ascending aorta. Which one of the following complications is least likely to occur with this type of dissection?

A. Aortic valve insufficiency

B. Haemopericardium

C. Mitral valve insufficiency

D. Cardiac tampondae

E. Myocardial infarction

A
  1. C. Mitral valve insufficiency

Dissection involving the ascending aorta can result in fatal complications, which include aortic rupture, cardiac tamponade, acute aortic regurgitation and acute myocardial infarction from dissection involving the coronary arteries. Involvement of the arch vessels results in morbidity from neurological complications.

50
Q
  1. A 38-year-old man with a 6-month history of dark coloured urine presents to his GP. He has a history of inflammatory bowel disease. LFTs are found to be deranged on blood work-up and he is then referred for an US liver. US shows non-specific coarsening of Liver echotexture suspicious for cirrhosis. A subsequent MRCP shows multiple focal strictures in the intrahepatic bile ducts alternating with normal calibre. What is the likely diagnosis?

A. Primary biliary cirrhosis

B. Primary sclerosing cholangitis

C. Haemochromatosis

D. Glycogen storage disease

E. Cholangiocarcinoma

A
  1. B. Primary sclerosing cholangitis

Primary sclerosing cholangitis (PSC) is an idiopathic, chronic, fibrosing inflammatory disease of the bile ducts that eventually leads to bile duct obliteration, cholestasis and biliary cirrhosis. A strong association with inflammatory bowel disease, especially ulcerative colitis, is noted (70% of cases). Although the cause of PSC is unknown, most experts believe it to be an autoimmune process because PSC may be associated with other autoimmune diseases such as retroperitoneal fibrosis, mediastinal fibrosis and Sjögren syndrome. The rate of progression is unpredictable, with up to 49% of symptomatic patients eventually developing biliary cirrhosis and liver failure. Treatment is usually palliative and includes medical therapy with orally administered agents such as ursodiol (ursodeoxycholic acid) or endoscopic or percutaneous mechanical dilation of dominant strictures. Cholangiographic findings usually include multifocal, intrahepatic bile duct strictures alternating with normal-calibre ducts, which sometimes produce a beaded appearance.

51
Q

@# 51. A 30-year-old woman who is 36 weeks pregnant is being evaluated with targeted ultrasound to investigate a recently noticed breast lump. Ultrasound images are reported to show a well-circumscribed, wider than tall, hypoechoic, solid mass corresponding to the abnormality. Follow-up imaging to assess stability shows progressive regression in size of the mass post-partum. What is the likely diagnosis?

A. Fibrocystic change

B. Fat necrosis

C. Inflamed intramammary node

D. Lactating adenoma

E. Breast hamartoma

A
  1. D. Lactating adenoma

Lactating adenomas are newly discovered painless lumps that appear during the third trimester or lactation. They are a freely mobile, homogenous hypoechoic or isoechoic mass with posterior acoustic enhancement (most common) and septa. The lesion regresses after breastfeeding.

52
Q
  1. A 17-year-old Afro-Caribbean boy presents with multiple painful soft-tissue masses around the right hip, some of which are ulcerating whitish fluid. Plain radiograph reveals a dense lobulated calcified mass with normal underlying bones at the right hip. His blood biochemistry is normal and there is no known history of renal or metabolic disease. What is the likely diagnosis?

A. Renal osteodystrophy

B. Lympocele with infection

C. Scleroderma

D. Burkitt’s lymphoma

E. Tumoural calcinosis

A
  1. E. Tumoural calcinosis

Tumoural calcinosis is a rare familial disease, typically presenting in young men of African origin, with progressive large nodular juxta-articular calcified soft-tissue masses. Patients have normal serum calcium and phosphorus and do not have any evidence of renal, metabolic or collagen-vascular disease.

The hips are the most frequently affected, followed by the elbows, shoulders and feet, and the disease is usually along the extensor surface of the joints.

The knees are almost never affected.

Occasionally, fluid fluid levels are seen with the calcium lying inferiorly, termed the sedimentation sign. The joints themselves are normal.

53
Q
  1. A 38-year-old man with back pain and sensory deficit in both lower limbs shows widening interpedicular distance on plain radiograph of the lumbar spine extending over four vertebral lengths. MRI reveals a hypointense rim around a well-defined lesion on T2W images, with intense enhancement on post-gadolinium images. What is the diagnosis?

A. Astrocytoma

B. Ganglioglioma

C. Neurofibroma

D. Ependymoma

F.. Subependymoma

A
  1. D. Ependymoma

Radiographs of patients with ependymomas may reveal scoliosis, canal widening with vertebral body scalloping, pedicle erosion or laminar thinning.
Ependymomas enhance intensely after intravenous administration of iodinated contrast material. Most spinal cord ependymomas are iso- or hypointense on T1-weighted images. Any hyperintensity suggests haemorrhage. They are hyperintense to the cord on T2-weighted images. About 20% 33% of ependymomas demonstrate the cap sign, a rim of extreme hypointensity (hemosiderin) seen at the poles of the tumour on T2-weighted images. This finding is secondary to haemorrhage, which is common in ependymomas and other highly vascular tumours (e.g., paraganglioma, haemangioblastoma) Most cases (60%) also show evidence of cord oedema around the masses. The average number of vertebral segments involved with abnormal signal intensity is 3.6. Cysts are a common feature, with 78%-84% of ependymomas having at least one cyst.
Subependymomas represent a variant of central nervous system (CNS) ependymomas. MR imaging findings are not sufficiently unique to enable the differentiation of ependymomas from
subependymomas; however subependymomas are eccentric, may not enhance and may not have surrounding oedema.

54
Q
  1. Chest X-ray of a boy shows shift of the heart and mediastinum to the right. There is also a tubular structure parallel to the right heart border with its maximum width close to the diaphragm. The finding suggests

A. ASD

B. Scimitar syndrome

C. Total anomalous pulmonary venous return

D. Intralobar sequestration

E. Inhaled foreign body

A
  1. B. Scimitar syndrome

Hypogenetic lung syndrome, also known as congenital venolobar syndrome or scimitar syndrome, is primarily a complex developmental lung abnormality with anomalous venous return. The most common features are lung hypoplasia, anomalous pulmonary venous return to IVC, pulmonary artery hypoplasia, bronchial anomalies and systemic arterial supply to hypoplastic lung. It almost always occurs on the right side and is slightly more common in women.
One constant component of this syndrome is an anomalous pulmonary vein or veins draining at least a part or the entire affected lung most commonly to the inferior vena cava just above or below the diaphragm. Uncommonly, the anomalous vein may drain into hepatic, portal, azygos veins; the coronary sinus; or the right atrium. A scimitar vein is a vertical curvilinear opacity in the right mid-lower lung, running along the right heart border inferomedially towards the diaphragm to join the IVC. A scimitar vein present on a frontal chest radiograph is called the scimitar sign.

55
Q
  1. A 67-year-old woman with 5.5 cm atherosclerotic abdominal aortic aneurysm is being worked up for a potential aortic endograft repair. All of the following are important imaging observations to be determined prior to treatment, except

A. Tortuosity of the aorta

B. Diameter of aortic aneurysms

C. Non-thrombosed residual lumen of the aneurysm

D. Flow characteristic at the aneurysm neck

E. Length of proximal and distal landing zones

A
  1. D. Flow characteristic at the aneurysm neck

An aneurysm occurs when a vessel diameter exceeds 1.5 times its normal size. In the abdomen this corresponds to 3 cm. These aneurysms should be repaired when the diameter exceeds 5-5.5 cm or the aneurysm expands more than 1 cm per year.
The greatest benefit of 3D volume-rendered imaging is the depiction and precise measurement of angulation in aneurysms with marked tortuosity.
The proximal landing zone consists of the region from the inferior-most renal artery to the beginning of the aneurysm. The maximal acceptable neck diameter is 32 mm. The length of the neck should be at least 15 mm (although one device allows a 7 mm neck). The angle between the superior portion of the aneurysm neck and the suprarenal aorta is preferably less than 60°.
Study results suggest higher complications for aneurysms larger than 5.5-6.5 cm. The shape can be described as saccular or fusiform. The residual lumen through the aneurysm should measure approximately 18 mm to allow passage and proper deployment of the device. The preferred distal landing zone is the common iliac artery. Evaluation is similar to that of the proximal neck with assessment of diameter, length, tortuosity and degree of calcification and thrombus. The common iliac artery diameter should not be larger than 25 mm, and at least 10 mm of length is required for an adequate .seal.

56
Q
  1. A 16-year-old girl presents to a gastroenterologist with intermittent episodes of diarrhoea, weight loss and abdominal discomfort over the past 2 years. A full blood count done shows macrocytic anaemia. She is referred for a barium small bowel follow-through examination to assess the small bowel The barium study reveals several discontinuous ileal strictures with alternate areas of dilatation. The terminal ileum is distinctly abnormal, with ulceration with ‘cobblestoning. What is the most likely diagnosis?

A. Ulcerative colitis

B. Coeliac disease

C. Whipple’s disease

D. Crohn’s disease

E. Tuberculosis

A
  1. D. Crohn’s disease

In Crohn’s disease involving either the small bowel or colon, broad linear ulcers that crisscross in longitudinal and transverse directions can produce a pseudopolypoid appearance usually termed cobblestoning.

57
Q
  1. With regard to mammography and MR! breast, which is incorrect?
A
  1. E. Needs IV contrast Does not need IV contrast

The advantages of mammograms include their reliable detection of calcification, lower cost than MRI, wide availability and their imaging speed, which is faster than MRI. The disadvantages include difficulty in presence of implants, discomfort caused by compression, difficulty in imaging dense breasts and the need to reposition the breast for different views. The advantages of MRI are its higher sensitivity, ability to be used with implants, good performance with dense breasts, not requiring compression, ability to image both breasts simultaneously and improved tissue resolution. The disadvantages of MRI are that it may not show calcification, is not good for DCIS, leads to many false-positive findings and is expensive.

58
Q
  1. A 52-year old man with painful hands and feet is referred for plain film imaging by his family doctor. Radiographs of both hands demonstrate bilateral resorption of the distal phalangeal tufts of the hands and feet. All the conditions below are differentials, except for

A. Psoriasis

B. Diffuse idiopathic skeletal hyperostosis

C. Scleroderma

D. Thermal injury

E. Raynaud disease

A
  1. B. Diffuse idiopathic skeletal hyperostosis (DISH)
    DISH is not a cause of acro-osteolysis.

Other causes of acro-osteolysis include leprosy, frost bite, electrical and thermal burns, PVC workers, sarcoidosis, rheumatoid arthritis, Reiter syndrome, pyknodysostosis, syringomyelia and hyperparathyroidism.

59
Q
  1. A 53 year-old woman with known retinal lesion undergoes an MRI of the spine in a regional neuroimaging centre. The MRI reveals a well-defined hyperintense lesion on T2W sequences with internal flow voids and intense enhancement in association with syringohydromyelia. What is the diagnosis?

A. Ependymoma

B. Astrocytoma

C. Haem angioblastoma

D. Lymphoma

E. Metastasis

A
  1. C. Haemangioblastoma

Haemangioblastomas manifest with diffuse cord expansion and variable signal intensity on T1-weighted images, most commonly isointense (50%) or hyperintense (25%) relative to the cord. On T2-weighted images, they have high signal intensity with intermixed focal flow voids. Surrounding oedema and a cap sign may be seen. Although up to 25% of haemangioblastomas may appear to be solid, cyst formation or syringohydromyelia is very common (up to 100% in some series). In fact, some cases may have the classic appearance of a cystic mass with an enhancing mural nodule characteristic of cerebellar haemangioblastomas. Contrast-enhanced imaging typically shows an intensely homogeneous enhancing tumour nodule.

60
Q
  1. A 6-year-old boy presents with increasing pain within his upper back, which came on insidiously over a few weeks. The child is otherwise well. A radiograph of his thoracic spine reveals collapse of the T9 vertebral body. The disc spaces are preserved; there is no kyphosis and no involvement of the posterior elements. Which of the following is the most likely diagnosis?

A. Ewings sarcoma

B. Metastasis

C Tuberculosis

D. Fracture

E. Langerhans cell histiocytosis

A
  1. E. Langerhans cell histiocytosis

The vast majority of vertebra plana lesions in relatively healthy children are caused by an eosinophilic granuloma of Langerhans cell histiocytosis. The other available options are all possible, but less common, differential diagnoses. There is usually preservation of the disc space and no kyphosis. The posterior elements are rarely involved.
Platyspondyly (multiple flat vertebral bodies, as opposed to vertebra plana) specially affecting lumbar vertebra by 2-3 years of age is a typical feature of Morquio syndrome.

61
Q

@# 61. A 72-year-old man with a 5.8 cm aortic aneurysm increasing at a rate of 1.2 cm every year is being worked up for an EVAR. Planning CT with multiformatted images and 3D volume-rendered reconstructed images have been obtained. All of the following measurements need to be provided at the MDT by the vascular radiologist, except

A. Size of aorta at the level of the renal artery

B. Size of aorta at the bifurcation

C. Largest diameter of the aneurysm

D. Size of the common iliac artery

E. Distance from aortic bifurcation to common iliac bifurcation

A
  1. A. Size of aorta at the level of the renal artery

Several important characteristics of the aneurysm must be accurately described for standard stent-graft sizing.

The aneurysm is described in terms of the proximal landing zone, the aneurysm sac, the distal landing zone and the vascular access.

Reported diameters should include the aorta at the level of the most inferior renal artery, the aortic neck 15 mm distal to the lowest renal artery, aorta at the bifurcation, the largest aneurysm sac diameter and the size of the common iliac arteries.

Additional measurements include the length of the aneurysm neck, the length from the lowest renal artery to the aortic bifurcation and the length of the aneurysm sac.

The length of the distal landing zone is described as the distance from the aortic bifurcation to the common iliac artery bifurcation.

Minimal diameters should be recorded in the distal landing zone and external iliac artery access vessels.

62
Q
  1. A 47-year-old admitted with acute diverticulitis becomes more unwell with swinging pyrexia right upper-quadrant pain, and acutely deranged LFTs. US liver show’s several ill-defined hypoechoic areas with very little posterior acoustic enhancement and some floating internal echoes. What is the diagnosis?

A. Hydatid disease

B. Liver metastasis

C. Multifocal HCC

D. Pyogenic liver abscess

E. Infected simple cysts

A
  1. D. Pyogenic liver abscess

Pyogenic abscesses, particularly when multiple, may be caused by haematogenous dissemination of either gastrointestinal infection via the portal vein or disseminated sepsis via the hepatic artery, ascending cholangitis or superinfection of necrotic tissue. Over one-half of liver abscesses are polymicrobic. Escherichia coli is the most common bacterium, but other anaerobic and aerobic organisms can be involved. Pyogenic abscesses demonstrate no sex predilection but most commonly involve middle aged patients.
On US, pyogenic micro-abscesses may manifest as either discrete hypoechoic nodules or ill-defined areas of distorted hepatic echogenicity. There may be little or no enhanced through transmission.
At contrast-enhanced CT, they appear as multiple small, well-defined hypoattenuating lesions. Faint rim enhancement and perilesional oedema can be observed, findings that help differentiate them from hepatic cysts.

63
Q
  1. Which of the following is malignant calcification in the breast?

A. Stellate

B. Pleomorphic

C Punctate

D. Curvilinear

E. Popcorn

A
  1. B. Pleomorphic

Calcifications are likely to be malignant if they are clustered rather than scattered throughout the breast and vary in size and shape (pleomorphic) and if they are found in a linear or ductal distribution. Benign calcifications could be coarse, ‘popcorn’ (fibroadenoma); ‘eggshell’ and curvilinear as in fat necrosis; ‘tramline’ (vascular), ‘broken needle’ or ‘lead pipe’ as in duct ectasia; punctate, stellate, and ‘teacup’ as in fibrocystic change.

64
Q
  1. Scaphoid waist fracture healing is often complicated by avascular necrosis, non-union, or delayed union. Which of the following is the primary reason for this?

A. The blood supply to the distal pole enters at the proximal pole.

B. The blood supply of the proximal pole enters at the waist.

G it is a difficult fracture to immobilise.

D. Fractures of the scaphoid are difficult to reduce.

E. Fractures are often comminuted.

A
  1. B. The blood supply of the proximal pole enters at the waist.

The blood supply to the proximal pole of the scaphoid enters at the waist and courses proximally. A waist fracture can therefore interrupt this tenuous blood supply, leading to avascular necrosis of the proximal pole or delayed/non-union. Non-displaced fractures are often treated conservatively, whereas displaced fractures usually require reduction and internal fixation. The scaphoid bone is the most commonly fractured carpal bone and tends to result from a fall on outstretched hand in a younger population rather than the common dorsally angulated distal radial fractures (Colles fracture). A high index of suspicion should be used when reporting trauma plain films of the wrist, and follow-up imaging following a period of conservative management should be employed when there is clinical concern but unremarkable presentation radiographs.

65
Q
  1. A 72-year-old man with a known malignancy undergoes a spinal MRI for characterisation of multilevel vertebral collapse identified on plain radiograph. All of the following are true about malignant vs osteoporotic causes of vertebral fractures, except

A. The involved vertebra are low on T1W images.

B. Paraspinal mass is useful in differentiating metastatic from benign fracture.

C. DWI can differentiate between malignant and benign compression fracture.

D. Posterior bulging of collapsed vertebral body suggests metastasis.

E. Acute osteoporotic fractures show intense enhancement post-contrast.

A
  1. B. Paraspinal mass is useful in differentiating metastatic from benign fracture.

Distinction between metastatic and acute osteoporotic compression fractures could be made on
the basis of MR imaging findings.
A convex posterior border of the vertebral body is more frequent in metastatic compression fractures than acute osteoporotic compression fractures. A higher frequency of abnormal signal intensity of the pedicle or posterior element has been observed in metastatic compression fractures. Epidural soft-tissue mass is suggestive of malignant vertebral collapse.
A paraspinal mass is not helpful in differentiation of the cause of vertebral collapse but is more commonly encountered in the setting of metastatic compression, where it is typically focal rather than diffuse. Signal intensity abnormalities in the marrow of vertebrae other than the collapsed vertebrae are more frequently seen in metastatic compression fractures than acute osteoporotic compression fractures.
Enhancement on post-contrast T1-weighted FS images is not useful in differentiation of acute osteoporotic fractures from malignant compression fracture, but it may be useful for old or chronic fractures, which will not show intense enhancement.
Moreover, on diffusion-weighted imaging (DWI) vertebral metastases with compression fractures can be safely distinguished from vertebra with benign compression fractures based on significantly different ADC- values.

66
Q
  1. A 4-year-old child presents with short stature and failure to grow. Plain radiographs reveal multiple abnormalities, including generalised increased density of long bones with thickened cortices, widened cranial sutures. Wormian bones, hypoplastic mandible
    and shortened pointed distal phalanges. Which of the following is the most likely diagnosis?

A. Pyknodysostosis

B. Osteopetrosis

C. Cleidocranial dysostosis

D. Osteosclerosis

E. Kinky hair syndrome

A
  1. A. Pyknodysostosis

Pyknodysostosis is a congenital abnormality that should be considered in the differential diagnosis of osteosclerosis. The patients arc typically short, have hypoplastic mandibles, widened cranial sutures, Wormian bones, brachycephaly, clavicular dysplasia, thick skull base and hypoplasia or non-pneumatisation of the paranasal sinuses. The distinguishing feature is aero-osteolysis with sclerosis. The distal phalanges appear as if they have been put in a pencil sharpener - they are pointed and dense.

67
Q
  1. An 83-year-old man with a history of bladder cancer and myocardial infarction was referred for radiofrequency ablation (RFA) because of his co-morbidities. All of the following are true regarding RFA, except

A. RFA uses high frequency alternating current to generate heat and high temperature to cause cell death.

B. Cell death occurs by denaturation of proteins (coagulative necrosis).

C. The tip of the electrode is placed in the centre of the lesion.

D. The heat sink effect results in a poor outcome in larger lesions.

E. Cell death starts at 49 degrees.

A
  1. C. The tip of the electrode is placed in the centre of the lesion.

In RFA, a high-frequency, alternating current with a wavelength of 460-500 kHz is emitted through an electrode placed within the targeted tissue. Grounding pads applied to the patient’s thighs complete the electrical circuit. Cell death starts at 49 degrees. Temperature above 60 degrees causes immediate cell death, and tissue charring occurs at 105 degrees. Cell death is induced by the denaturation of proteins.
For percutaneous imaging-guided RFA, the energy is delivered into the target tissue by means of needle like electrodes. Unlike in a typical biopsy, the electrode tip should be advanced to the deep margin of the tumour.
On follow-up CT, ablated tumours often have internal areas of increased attenuation or increased signal intensity at CT and MRI. Areas of contrast enhancement (>10 HU or >15% with CT and MRI, respectively) are indicative of residual viable RCC. Residual viable tumour can be treated with additional ablation sessions.
Heat-sink phenomena refers to the reduction in tissue temperature due to the conductive effects of adjacent vessels or airways. It is an explanation for distortion of the ablation zone and poor outcome in larger lesions. The heat-sink effect can be overcome by pharmacologically reducing blood flow, intra-arterial embolisation, intravascular balloon occlusion, Pringle manoeuvre or reducing treatment zone.

68
Q
  1. A 76 year-old woman with anorexia, weight loss, and obstructive LFTs worsening over a period of 6 months presents to a gastroenterologist. She has been referred for an MRCP, which shows extensive dilatation of the intrahepatic bile ducts secondary to a long segment stricture at the proximal CBD. Distal CBD and pancreatic duct are normal. The most likely diagnosis is

A. Post-infective stricture

B. Cholangiocarcinoma

C. Intraductal stones

D. Mirizzi syndrome

E. Ampullary carcinoma

A
  1. B. Cholangiocarcinoma

Intrahepatic cholangiocarcinoma is the second most common primary hepatic tumour. Various risk factors have been reported for intrahepatic cholangiocarcinoma, and the radiologic and pathologic findings of this disease entity may differ depending on the underlying risk factors. Intrahepatic cholangiocarcinoma can be classified into three types on the basis of gross morphologic features: mass-forming (the most common), periductal infiltrating and intraductal growth.
There are a number of recognised risk factors for cholangiocarcinoma that all share the common feature of chronic biliary” inflammation: liver flukes (Opisthorchis viverrini, Clonorchis sinensis), hepatolithiasis (recurrent pyogenic cholangitis), PSC, viral infection (human immunodeficiency virus, hepatitis B virus, hepatitis C virus, Epstein-Barr virus), anomalies and malformations (anomalous pancreaticobiliary junction and choledochal cyst, fibrocystic liver diseases, such as Caroli disease), environmental or occupational toxins (Thorotrast, dioxin, polyvinyl chloride), biliary tract enteric drainage procedures and heavy alcohol consumption

69
Q
  1. A 53-year old woman presents to the breast clinic with a palpable left axillary node.
    Biopsy reveals adenocarcinoma, likely secondary to breast primary. Clinical examination is unremarkable. This is followed by a normal ultrasound and a mammogram, which was interpreted to be normal. MRI demonstrated a 1.5 cm slightly irregular heterogeneously enhancing lesion in the left breast and a second smaller enhancing nodule. What management should the reporting radiologist suggest?

A. Likely benign disease. Follow-up imaging in 6 months.

B. Ignore. Routine follow-up.

C. Masses only visible on MRI and not on mammogram; does not require biopsy.

D. Suspicious masses need biopsy.

E. Advanced disease. Straight to mastectomy.

A
  1. D. Suspicious masses need biopsy.

Although the use of contrast-enhanced MRI of the breast has increased both the sensitivity and the specificity of breast cancer detection, common causes of false-positive and rarer causes of false-negative diagnoses still occur. Irregular heterogeneous enhancement is not typical of a benign lesion, hence biopsy is required. In addition, note that cancers can present with smooth well-defined margins, hence an apparent benign appearance in a patient with BRCA1 mutation should with be regarded with caution and may need biopsy.

70
Q
  1. A 24-year-old man presents to his family doctor with chronic hindfoot pain. Plain radiographs reveal elongation of the anterior dorsal calcaneus on lateral projection of the foot, often described as anteater’s nose. What is the diagnosis?

A. Talocalcaneal coalition

B. Calcaneonavicular coalition

C. Talonavicular coalition

D. Calcaneocuboid coalition

E. Cubonavicular coalition

A
  1. B. Calcaneonavicular coalition

Tarsal coalition is best confirmed with CT. CT and MRI can show direct bony continuity (osseous coalition) or fluid/cartilage intensity on MR/irregular serrated articular surface in case of cartilaginous coalition.
A calcaneonavicular coalition shows the classic anteater’s nose appearance on lateral radiograph.
The C sign is seen on a lateral radiograph in a patient with talocalcaneal coalition. Prominent talar beak is also associated with talocalcaneal coalition. Both calcaneonavicular and talocalcaneal coalition are equally common.

71
Q
  1. All of the following tumours are associated with drop metastasis, except

A. Medulloblastoma

B. PNF.T

C. Ependymoma

D. Melanoma

E. Pineocytoma

A
  1. D. Melanoma

Drop metastasis refers to CSF seeding of intracranial neoplasm. Post-contrast images show ‘sugar coating’ of the brain and spinal cord in patients with leptomeningeal drop metastases or leptomeningeal carcinomatosis.
It can occur secondary to CNS involvement by distant primary tumours as well as primary CNS neoplasms.
CNS tumours with drop metastasis include PNET, medulloblastoma, anaplastic glioma, ependymoma, germinoma, pineoblastoma, pineocytoma and rarely choroid plexus carcinoma and angioblastic meningioma. Non-CNS primaries include breast, lung, melanoma and lymphoma.

72
Q
  1. A female child with short stature presents with gradual onset hearing loss. Diagnostic work-up reveals micromelic dwarfism, diffuse demineralisation and thinning of cortical bone, mild scoliosis, and old fractures of the vertebral bodies and long bones. There is evidence of poor dentition. Which of the following is the most likely diagnosis?

A. Hypophosphatasia

B. Osteogenesis imperfecta

C. Paget’s disease

D. Osteoporosis

E. Achondroplasia

A
  1. B. Osteogenesis imperfecta

Osteogenesis imperfecta is an inherited disorder that results from mutations in either the COL1A1 or COL1A2 gene of Type 1 collagen. The disease is usually apparent at birth or in childhood, but more mild forms of the disease may not be apparent until adulthood. The disease is classified into Types I-IV, with Type I, the mildest form, being described above. The presenile hearing loss is caused by otosclerosis. The differential diagnosis can be resolved by the extra-skeletal manifestations (blue sclerae and dentinogenesis imperfecta). The other types are Type II (lethal perinatal). Type III (severe progressive) and Type IV (moderately severe).

73
Q
  1. A 66-year-old man with hepatocellular carcinoma is being worked up for transcatheter arterial chemoembolisation (TACE) of the liver. All of the following are correct regarding TACE of the liver, except

A. TACE is useful in patients with Child-Pugh Class C Cirrhosis.

B. It is essential to assess patency of the portal vein.

C. Biliary surgery increases complications.

D. Normal liver receives only a third of its blood from the hepatic artery.

E. Diffuse involvement of the liver results in a poor outcome.

A
  1. A. TACE is useful in patients with Child-Pugh Class C cirrhosis.

TACE combines the effect of targeted chemotherapy with the effect of ischaemic necrosis induced by arterial embolisation. TACE provides a survival benefit in primary hepatocellular carcinoma (HCC) based on randomised controlled studies. The normal parenchyma of the liver receives two-thirds of its necessary blood supply from the portal vein and the remaining one-third from the hepatic artery. However, it is well known that vascularisation of HCC is mostly dependent on the hepatic artery. TACE is not done for patients with a severely compromised liver function such as Child Pugh classification C or late B. Though superselective TACE may be attempted in a patient with compromised liver function, if the patient has a diffuse or massive HCC or an HCC involving the major portal veins, this precludes the practice of safe TACE.
The underlying factors associated with increased procedural complication include compromised liver function, main portal vein obstruction, biliary tract obstruction and a previous history of bile duct surgery.

74
Q

@# 74. A 67-year-old man with a family history of colorectal cancer and a background history of COPD has been referred for a screening barium enema, which shows multiple small filling defects throughout the descending and sigmoid colon but no colonic mass. The most likely diagnosis is

A. Hereditary non-polyposis syndrome

B. Ulcerative colitis

C. Crohn’s disease

D. Colonic serosal metastasis

E. Pneumatosis coli

A
  1. E. Pneumatosis coli

Pneumatosis is the presence of gas bubbles within the wall of the involved segment of bowel. It is seen in a wide variety of conditions. It is widely divided into two groups: primary (idiopathic) and secondary. Conditions associated with secondary pneumatosis include obstruction, pulmonary disease such as COPD and asthma, vascular conditions such as ischaemia and infarction, inflammatory conditions such as Crohn’s and UC, necrotising enterocolitis, drugs such as steroids and chemotherapy, collagen vascular diseases such as scleroderma, SLE and dermatomyositis.

75
Q
  1. A 42-year-old man with known ankylosing spondylitis presents to the A&E department with blood in the urine and right loin pain. CT urogram performed out of hours showed calyceal horns and signet ring appearance on excretory phase study. The most likely cause would be

A. Renal TB

B. Acute pyelonephritis

C. Acute tubular necrosis

D. Right renal stone

E. Renal papillary necrosis

A
  1. E. Renal papillary necrosis

Urographic findings during this period of early ischaemic change are usually normal. On CT, the early ischaemic changes are best depicted on nephrographic phase scans as small, poorly marginated areas of diminished enhancement at the tip of the medullary pyramid.

Calyceal deformities in renal papillary necrosis occur in three forms:

medullary (round or oval cavity, calyceal blunting),

papillary (triangular cavity, ‘lobster daw’ appearance)

and in situ.

If the papilla detaches completely, a typical ring-like shadow is produced by the necrotic papilla in a contrast material-filled cavity; if the sloughed papillae remain in situ there is a ‘signet ring* appearance.

76
Q
  1. A 44-year-old man presents to the A&E department with acute exacerbation of knee pain and recurrent episodes of locking. Plain radiograph performed in the A&E department showed multiple calcified/ossified loose bodies in the suprapatellar pouch and behind the knee with underlying tricompartmental OA (osteoarthritis) like changes. What is the diagnosis?

A. CPPD

B. Primary’ synovial osteochondromatosis

C. Secondary synovial osteochondromatosis

D. Gout

E. Haemophilia

A
  1. C. Secondary synovial osteochondromatosis

Primary synovial osteochondromatosis is a benign self-limiting monoarticular disorder characterised by proliferation and metaplastic transformation of the synovium with formation of multiple intra articular cartilaginous or osteocartilaginous loose bodies. The form is not associated with OA, whereas the secondary form is associated with articular surface disintegration.
Classic radiographic features include multiple loose intra-articular bodies in varying grades of mineralisation. MRI shows varying grades of intermediate to high T1 signal to the loose bodies depending on the amount of mature osseous element (cf. pigmented villonodular synovitis, which shows low signal debris/areas on T2-weighted images; lipoma arborescens, which shows metaplastic frond-like synovial fat with high signal on T1-weighted images; and synovial haemangioma with mixed features).

77
Q
  1. A 63-year-old man with worsening bilateral leg pain has been sent for an MRI. The MRI report stated secondary spinal canal stenosis at multiple levels involving the lumbar spine. All the following are causes of acquired canal stenosis, except

A. Ligamentum flavum hypertrophy

B. Facet joint hypertrophy

C. Epidural lipomatosis

D. Congenital short pedicles

E. Disc herniation

A
  1. D. Congenital short pedicles

Spinal stenosis is defined as encroachment of the spinal canal or lateral recess by bone or soft- tissue components.
Congenital short pedicles are idiopathic or developmental, secondary to achondroplasia, hypochondroplasia, Down’s syndrome and Morquio disease. Acquired causes include hypertrophy of ligamentum flavum, facet joint OA, disc protrusion, spondylolysis or spondylolisthesis, surgical fusion, fracture (benign or metastatic), Paget’s disease, epidural lipomatosis and ossification of the posterior longitudinal ligament.

78
Q
  1. A 10-year-old girl of Jewish descent presents with pain in her left thigh. A radiograph reveals diffuse medullary osteoporosis, a ‘flask shaped’ distal femur, a serpentine area of sclerosis within the femoral metaphysis, and a sharply circumscribed endosteal lytic lesion in the distal femur with a pathological fracture. Clinical examination reveals splenomegaly. Which of the following is the most likely diagnosis?

A. Thalassaemia

B. Osteopetrosis

C. Diaphyseal aclasis

D. Gaucher disease

E. Rickets

A
  1. D. Gaucher disease

Although the Ashkenazi Jews are particularly predisposed to this hereditary condition, Gaucher disease is not confined to any particular ethnic group or sex. Splenic enlargement is detected in up to 95% of cases. There is abnormal modelling of the distal femur and proximal tibia secondary to marrow infiltration, leading to an ‘Erlenmeyer flask’ deformity. However, this feature is not diagnostic for Gaucher disease and may be seen in all the other answer options. Diffuse medullary osteoporosis, bone infarcts and sharply circumscribed endosteal lytic lesions (owing to marrow replacement) can also be seen. The combination of Jewish ancestry, the radiographic features described and splenomegaly makes Gaucher disease the most likely diagnosis.

79
Q
  1. All of the following statements regarding post-embolisation syndrome following a TACE procedure are true, except

A. It depends on the histology of the tumour being treated.

B. It is caused by tumour necrosis or damage to liver tissue.

C. There is elevation of hepatic transaminase levels.

D. Abdominal pain, fever, malaise and vomiting are reported.

E. It does not prolong hospital stay.

A
  1. A. It depends on the histology of the tumour being treated.

The most common complication of TACE is post-embolisation syndrome, which consists of transient abdominal pain and fever occurring in 60% 80% of patients after TACE. Elevation of the level of hepatic transaminases typically accompanies post-embolisation syndrome. Whether post-embolisation syndrome reflects damage to the normal liver parenchyma or tumour necrosis is uncertain. Though prolonged hospitalisation may be required to monitor a patient and to control abdominal pain, vomiting and/or malaise, post-embolisation syndrome is self limiting within 3-4 days, and the use of antibiotics is not necessary to treat the fever.
Hepatic failure after TACE is related to TACE-induccd ischaemic damage to the non-tumorous liver tissue. Several risk factors have been identified, including portal vein obstruction, use of a high dose of anticancer drugs and Lipiodol, a high level of bilirubin, prolonged prothrombin time and advanced Child-Pugh class.
Other TACE-related complications occur in less than 10% of treatment sessions and include ischaemic cholecystitis, hepatic abscesses and biliary strictures. Development of a liver abscess has been linked to previous intervention in the biliary system being prone to an ascending biliary infection. Upper gastrointestinal complications such as gastritis, ulceration and bleeding can occur by the regurgitation of embolic agents into the gastric arteries (anatomical variants), which should be recognised on pretreatment imaging.

80
Q
  1. A 45-year-old man with a known history of a polyposis syndrome presents to the emergency department with abdominal distention and bloating. A CT scan of the abdomen and pelvis reveals a large soft tissue density mass within the small bowel mesentery displacing the bowel loops and compressing the right ureter with resultant hydronephrosis. Which of the following options describes the likely tumour and underlying polyposis syndrome?

A. Desmoid tumour – Gardners syndrome

B. Desmoid tumour Turcot syndrome

C. Desmoid tumour - Cronkhite-Canada syndrome

D. Carcinoid syndrome

E. Desmoid tumour MEN syndrome

A
  1. A. Desmoid tumour-Gardner’s syndrome

Aggressive fibromatosis, or intra-abdominal Desmoid tumour, represents a benign proliferative process that has a tendency to locally recur. Mesenteric involvement is more often seen in cases related to typical familial adenomatous polyposis syndrome (PAPS) (Gardner syndrome).
At CT, the margins of these lesions may appear irregular or smooth. The specific diagnosis of a mesenteric Desmoid tumour would be strongly suggested by a known diagnosis of FAPS or its associated CT findings, such as colonic polyps or masses, previous total colectomy, and polyps or masses involving the duodenum, stomach or periampullary region.

81
Q
  1. All the following are correct regarding the use of MR urography (MRU) except

A. Cine MRU is useful for assessing stenosis.

B. Diuretic administration improves the quality of MR excretory urogram.

C. Diuretic administration reduces the time during which images can be obtained.

D. Motion suppression is critical for MR urography.

E. Oral hydration can reduce quality due to fluid-filled bowel loops on T2 FSE.

A
  1. C. Diuretic administration reduces the time during which images can be obtained.

T2-weighted techniques were the first clinically relevant means of visualising the urinary tract
with MRI. Static-fluid MR urography does not require the excretion of contrast material and is therefore useful for demonstrating the collecting system of an obstructed, poorly excreting kidney. Static-fluid MR urograms can be obtained in 1-2 seconds, which allows multiple images to be obtained sequentially in a short period of time and played as a cine loop. Cine MR urography is particularly helpful in confirming the existence of urinary tract stenosis. For patients with non- dilated systems, the use of hydration, diuretics or compression may enhance the quality of MR urography. In case of excretory MR urogram, a gadolinium-based contrast agent is administered intravenously, and the collecting systems are imaged during the excretory phase. Gadolinium shortens the T1 relaxation time of the urine, allowing the urine to initially appear bright on T1-weighted images. Diuretic administration can improve the quality of excretory MR urography by enhancing urine flow, resulting in dilution and uniform distribution of gadolinium based contrast material throughout the urinary tract; it also expands the temporal window during which images can be obtained. The primary imaging sequence for excretory MR urography is the 3D gradient-echo sequence. Fat suppression enhances the conspicuity of the ureters and is recommended. Motion suppression is critical for MR urographic sequences, and breath hold acquisitions have been shown to better demonstrate the pelvicaliceal systems compared with respiratory triggering. IV hydration is preferred to oral hydration because fluid-filled structures can interfere with imaging on T2 sequences.
MR urography is more sensitive and specific for non-calculous urinary tract obstruction than unenhanced CT. CT is more sensitive for stone disease. MRU is also used for evaluation of congenital urinary tract anomalies and haematuria, whereas the role of MRU for screening patients at risk for urothelial malignancy has yet to be defined.

82
Q

@# 82. When performing an M R arthrogram, what is the correct concentration of gadolinium (Gd-DTPA, Magnovist) that should be injected into the joint prior to imaging?

A. 0.01 mmol/L

B. 0.1 mmol/L

C. 2.0 mmol/L

D. 20 mmol/L

E. 200 mmol/L

A
  1. C. 2.0 mmol/L

It is important when performing MR arthrography that the correct concentration of gadolinium is used in order to achieve the best arthrographic result. Too concentrated or too dilute a solution will result in a suboptimal study. A dose of 2 mmol/L is the recognised concentration of Gd-DTPA; this can be achieved by adding 0.8 mL of neat Gd-DTPA to 100 mL of normal saline. An alternative method would be to inject 4 mL of Gd-DTPA into a 500 mL bag of normal saline to achieve the same 2 mmol/L concentration. It is also important to avoid inadvertent intra-articular injection of air, as this will result in susceptibility artefact at the subsequent MRI that may limit its diagnostic accuracy.

83
Q
  1. Plain radiograph of a 9-month-old baby girl shows a large soft-tissue mass in the pelvis with punctate calcification. MRI reveals a large, tabulated, sharply demarcated tumour with extremely heterogeneous signal on T1W images. What is the diagnosis?

A. Anterior sacral meningocoele

B. Sacrococcygeal teratoma

C. Caudal regression syndrome

D. Rhabdomyosarcoma

E. Rectal duplication

A
  1. B. Sacrococcygeal teratoma

Sacrococcygeal teratoma is the most common presacral germ cell tumour in children and the most common solid tumour in neonates. The benign form accounts for 60% of all sacrococcygeal teratomas.
Benign teratomas are predominantly cystic; have attenuation similar to fluid on CT; and may include bone, fat and calcification. Cystic areas appear low on T1-weighted and high on T2-weighted MRI. Fatty tissue demonstrates high signal intensity on T1-weighted images, whereas calcification is depicted as a signal void. The coccyx is always involved, even in benign sacrococcygeal teratoma, and must be resected with the tumour. Malignant teratomas are more solid, and haemorrhage and necrosis are common. Approximately 50% of benign teratomas contain calcification, whereas it is seldom seen in malignant tumours. Malignant teratomas may metastasise.
Anterior sacral meningocoele is a congenital abnormality that arises from herniation of the CSF-filled dura mater through a sacral foramen or a defect in the sacral bone. Eccentric defect in sacrum results in a scimitar appearance on plain film.

84
Q

@# 84. A skeletal survey was performed on a 2-year-old boy with short stature. The lateral film of the spine revealed abnormal vertebral bodies with a central anterior ‘beak’ and generalised flattening. Radiographs of the hands showed a pointed proximal fifth metacarpal base with a notch at the ulnar aspect. Which of the following is the most likely diagnosis?

A. Hunter syndrome

B. Hurler syndrome

C. Morquio syndrome

D. Achondroplasia

E. Nail patella syndrome

A
  1. C. Morquio syndrome

The mucopolysaccharidoses are a group of inherited diseases characterised by abnormal storage and excretion in the urine of various mucopolysaccharides.

Patients with these diseases have short stature and characteristic plain film findings.

A characteristic finding in the hands is a pointed proximal fifth metacarpal base that has a notched appearance to the ulnar aspect.

There is generalised flattening of the vertebral bodies (platyspondyly).

Hunter and Hurler syndromes demonstrate an anterior vertebral beak that is inferiorly positioned, whereas Morquio syndrome demonstrates an anterior vertebral beak that is centrally positioned.

Although achondroplasia can cause rounded anterior beaking in vertebra of the upper lumbar spine, the findings described within the hands are more typical of the mucopolysaccharidoses.

85
Q

@# 85. A middle-aged woman underwent uterine fibroid embolisation (UFE) recently. All of the following statements regarding prognosis are correct, except

A. The enhancement pattern correlates well with treatment response.

B. Pedunculated fibroids with narrow pedicle are unfavourable.

C. Fibroids more than 15 cm in size may continue to give bulk symptoms.

D. Progressive liquefaction of fibroid post-treatment results in a high signal on T2W images.

E. Fibroids with high T1 signal pretreatment respond better to UFE.

A
  1. E. Fibroids with high Tl signal pretreatment respond better to UFE.

UFE is a minimally invasive treatment for uterine fibroids. MRI should be used to evaluate patients before and after UFE to accurately assess fibroid location within the uterus.
fibroid number and size, and the presence or absence of fibroid enhancement on contrast material-enhanced images.
Absolute contraindications include pregnancy, known or suspected gynaecologic malignancy, and current uterine or adnexal infection; relative contraindications include contrast material allergy, coagulopathy and renal failure. Pedunculated subserosal fibroids with a narrow stalk (<2-3 cm) are a relative contraindication to UFE because of the potential risk of detachment. Cervical fibroids tend to respond less favourably. The maximum size threshold for embolisation is 13-15 cm. Above this, the post-embolisation volume may still result in bulk symptoms, and the necrosis from a large fibroid may result in a protracted post embolisation syndrome.
Fibroids with increased cellular content and degenerate fibroids may demonstrate a high signal on T2-weighted MRI; however, a heterogeneous or markedly hyperintense T2 signal suggests degeneration. Completely hyalinised fibroids have low T2. T1 hyperintensity pre-MRI suggests fatty or haemorrhagic/red degeneration and is a negative predictor of success with a lower reduction in vascularity compared to fibroids with a low T1 signal.
After successful embolisation, fibroids may undergo progressive liquefaction with increasing T2 signal. Volume reduction is greater in T2 hyperintense fibroids and hypervascular fibroids compared to hypovascular ones. Persistent enhancement post embolisation is sign of incomplete fibroid infarction. An increase in signal on T1-weighted images is typically observed immediately after embolisation.

86
Q
  1. A 66-year-old woman with chronic right upper-quadrant pain, anorexia and weight loss is referred by her CP for a CT scan of her abdomen and pelvis. The scan shows an irregular hypodense mass replacing the gallbladder, with infiltration into the surrounding liver
    and enlarged periportal nodes. The following are risk factors for gallbladder carcinoma, except

A. Chronic gallbladder inflammation

B. Porcelain gallbladder

C. Choledochal cyst

D. Cirrhosis

E. Ulcerative colitis

A
  1. E. Ulcerative colitis

Gallbladder carcinoma is highly lethal, as anatomic factors promote early local spread. The ease with which this tumour invades the liver and surrounding structures, including the biliary tree, contributes to its high mortality. The median survival is 6 months, indicating that the majority of patients present with advanced disease.
Epidemiologic studies have shown that female sex, age, postmenopausal status and cigarette smoking are risk factors. Ethnic origin, increased body mass and physician-diagnosed typhoid arc risk factors in the high-incidence populations of La Paz, Bolivia, and Mexico City, Mexico. Exposure to chemicals used in the rubber, automobile, wood finishing and metal fabricating industries has been associated with an increased risk of gallbladder carcinoma. Cholelithiasis is a well-established risk factor for the development of gallbladder carcinoma, and gallstones arc present in 74%-92% of affected patients. Gallstones cause chronic irritation and inflammation of the gallbladder, which leads to mucosal dysplasia and subsequent carcinoma. Porcelain gallbladder is an uncommon condition in which there is diffuse calcification of the gallbladder wall, and 10%-25% of patients with this condition have gallbladder carcinoma.
Several pathologic and congenital anatomic anomalies are associated with a higher prevalence of gallbladder carcinoma, compared with that in the general population. These conditions include congenital cystic dilatation of the biliary tree, choledochal cyst, anomalous junction of the pancreaticobiliary ducts (with or without a coexistent choledochal cyst) and low insertion of the cystic duct
The cross-sectional imaging patterns of gallbladder carcinoma have been described as a mass replacing the gallbladder in 40%-65% of cases, focal or diffuse gallbladder wall thickening in 20%-30% and an intraluminal polypoid mass in 15%-25%.

87
Q

@# 87. A 23-year-old woman with large bilateral low density renal lesions on CT is also known to have a large posterior fossa brain tumour. Review of the old notes reveals that she has
had several visits to the ophthalmology department. The condition that she is most likely to be
suffering from is

A. Tuberous sclerosis

B. Von Hippel-Lindau syndrome

C. Wunderlich syndrome

D. NF1

E. No syndrome the findings are unrelated

A
  1. B. Von Hippel-Lindau syndrome

Von Hippel-Lindau (VHL) disease is a rare, inherited, multisystem disorder that is characterised by development of a variety of benign and malignant tumours. The spectrum of clinical
manifestations of the disease is broad. These include retinal and CNS haemangioblastomas (mostly affecting cerebellum/posterior fossa), endolymphatic sac tumours, renal cysts and tumours (renal cell carcinoma), pancreatic cysts and tumours (serous cystadenoma, adenocarcinoma and neuroendocrine tumours), pheochromocytomas, and epididymal cystadenomas.
Retinal haemangioblastomas are among the most frequently and earliest detected VHL disease lesions.

88
Q
  1. A 32-year-old runner presents to the sports clinic with acute exacerbation of right heel pain. He is referred for an MRI study to investigate a potential underlying cause. The MRI reveals a poorly defined hypointense lesion inferior to the calcaneum on both T1W and T2W sequence. A further smaller lesion is noted beneath the first metatarsal head. There is marked contrast enhancement on the post-gadolinium sequences. What is the likely diagnosis?

A. Haemangioma

B. Mortons neuroma

C. Plantar fibromatosis

D. Abscess

E. Neurofibroma

A
  1. C. Plantar fibromatosis

Plantar fibromas are benign fibrous nodules commonly affecting the medial plantar fascia, with typical anatomical location, ultrasound and MRI findings. Plantar fibromas are well defined and hypoechoic, appearing along the plantar fascia on ultrasound; some are vascular on Doppler. On MRI they are mainly low on T1-weighted and T2-weighted sequences, although sometimes T2-weighted signal is high secondary’ to an increased cellular component. Variable enhancement post-gadolinium is noted. They are bilateral in 10%-25%, and association with Dupuytren’s contracture, palmar fibromatosis and Peyronie’s disease has been described.

89
Q

@# 89. An unenhanced CT brain performed on a young man with sudden severe occipital headache shows acute subarachnoid haemorrhage with most of the blood at the foramen magnum. Considering that the source is a ruptured aneurysm, what is the most likely location for the aneurysm in this patient?

A. ACOM

B. PCOM

C. ACA

D. AICA

E. PICA

A
  1. E. PICA

The location of blood in cases of subarachnoid haemorrhage from a ruptured aneurysm can pinpoint the site of aneurysm in 70% of cases.

90
Q

@# 90. A 4-year-old girl presents with progressive enlargement of her right thigh, with episodes
of unprovoked bleeding from pigmented lesions over her right thigh, which have been present since birth. A lower limb venogram of the right kg demonstrates absence of the deep venous system, with varicose veins on the lateral aspect of the right leg. Which of the following is the most likely diagnosis?

A. Klippel-Trenaunay syndrome

B. Neurofibromatosis

C. Beckwith-Wiedemann syndrome

D. Macrodystrophia lipomatosis

E. Maffuci syndrome

A
  1. A. Klippel-Trenaunay syndrome

Klippel-Trenaunay syndrome is a sporadic, rare, mesodermal abnormality that usually affects a single lower limb. It is characterised by a triad of a port wine naevus (unilateral cutaneous capillary haemangioma often in a dermatomal distribution on the affected limb), overgrowth of distal digits/entire extremity (involving soft tissue and bone) and varicose veins on the lateral aspect of the affected limb. Although the other options can produce limb hypertrophy, they would not be expected to show all the features of the triad described.

91
Q
  1. Regarding uterine artery embolisation (UAE) for treatment of uterine fibroids, all of the following are correct, except

A. MRI is used to delineate arterial anatomy prior to treatment.

B. PVA or embospheres are used to embolisc a fibroid.

C. Gel foam or coils are preferred in post partum haemorrhage.

D. Endometritis shows signal void in the uterine cavity on all sequences.

E. Ovarian dysfunction after UAE is more common in women under 45 years.

A
  1. E. Ovarian dysfunction after UAE is more common in women under 45 years.

MRI with contrast is required to delineate uterine and ovarian artery anatomy prior to
treatment. In most cases, bilateral UAE is needed, because most uterine fibroids, whether single or multiple, receive blood supply from both uterine arteries. Women over 45 years of age have been shown to have a higher prevalence of uterine-ovarian arterial anastomoses and are at increased risk for ovarian dysfunction after UAE.
Although infrequent, major adverse events can occur and include ovarian failure or amenorrhea, fibroid expulsion and rarely venous thromboembolism. Intracavitary fibroids are more likely to be expelled. Ischaemia leading to uterine necrosis, arterial dissection, endometritis and fibroid infarction are other complications. On MRI, endometritis may manifest as uterine enlargement with T1 hyperintense intracavitary haematoma; gas appears as a signal void on all sequences. Contrast enhancement may increase the conspicuity of intracavitary fluid collections.
Embolic agents used for fibroids include PVA (300-350 microns) or embospheres, whereas agents used for postpartum haemorrhage are gel foam particles, coils (occasionally) or n-butyl-cyanoacrylate (glue).

92
Q
  1. A 62-year-old man with known hepatocellular carcinoma on a background oflong-standing liver cirrhosis is scheduled to have a TACE procedure. Which one of the following is an absolute contraindication to TACE therapy for hepatocellular carcinoma in a cirrhotic patient?

A. Contrast medium allergy

B. Replacement of 25% of the liver by the tumour

C. Total bilirubin greater than 2 mg/dL

D. Biliary tree obstruction

E. Child-Pugh Class C cirrhosis

A
  1. E. Child-Pugh Class C cirrhosis

Absolute and relative contraindications for conventional TACE in patients with HCC are as follows.
Absolute contraindications:
* Decompensated cirrhosis (Child-Pugh C or higher)
* Jaundice
* Clinical encephalopathy
* Refractory ascites
* Extensive tumour with massive replacement of both lobes
* Severely reduced portal vein flow
* Technical contraindications to hepatic intra-arterial treatment
* Renal insufficiency (creatinine clearance <30 mL/min)
Relative contraindications;
* Tumour size >10 cm.
* Co-morbidities involving compromised organ function such as cardiovascular and lung disease.
* Untreated varices present a high risk of bleeding.
* Bile duct occlusion or incompetent papilla due to stent or surgery.

93
Q
  1. A 33-year-oid woman on immunosuppressant medication presents to the urology clinic with haematuria. Review of old notes reveals a history of treated pulmonary tuberculosis. All the findings suggest renal/ureteric TB, except

A. Hydrocalyx with infundibular stenosis

B. Amorphous parenchymal calcification

C. Sawtooth ureter

D. Bilateral involvement in 75%

E. Distal ureteric calcification

A
  1. D. Bilateral involvement in 75%

The GU tract is the second most common site after lungs.

A single kidney is involved in 75% of cases. Radiological features include TB pyonephrosis (putty kidney), small shrunken kidney with dystrophic or amorphous calcification (autonephrectomy), irregular eroded calyx (moth-eaten appearance), cavities connected to the collecting system, hydrocalycosis and infundibular strictures, phantom and amputated calyces, kink of renal pelvis, and renal stones.

Strictures in the ureter result in sawtooth, corkscrew, beaded and pipestem ureter.

Calcification is seen in the distal ureter, whereas bladder wall calcification is rare.

Bladder TB results in thick-walled small volume thimble’ or shrunken bladder.

Prostate TB causes radiating, streaky, low-signal areas on T2-weighted images (watermelon sign).

94
Q
  1. A 60-year-old man complaining of pain at multiple sites for several months is referred for skeletal evaluation. A whole-body bone scan demonstrates diffusely increased uptake in the axial skeleton, with minimal uptake in the appendicular skeleton, soft tissues and kidneys. Which of the following is the most likely diagnosis?

A. Diffuse bone metastasis

B. Hypoparathyroidism

C. Hypothyroidism

D. Osteoporosis

E. Normal physiological uptake

A
  1. A. Diffuse bone metastasis

The findings of the bone scan are those of a so-called super scan. This is when a bone scan reveals a strikingly high radiotracer uptake within bone, compared with soft tissues, and diminished to absent renal uptake. A variety of diseases that cause diffusely increased bone turnover can demonstrate this picture. Widespread bone lesions, such as diffuse skeletal metastases, are the most frequent cause, but other causes include myelofibrosis, aplastic anaemia, leukaemia and widespread Paget disease. Metabolic causes include renal osteodystrophy, osteomalacia, hyperparathyroidism and hypothyroidism.

Unlike in metastatic disease, the uptake in metabolic bone disease is more uniform in appearance and extends to the distal appendicular skeleton.

95
Q
  1. A 65 year-old man undergoing a CT brain for right-sided weakness shows a unilocular thin-walled extra-axial CSF density lesion replacing the anterior portion of the right temporal lobe. No calcification or contrast enhancement is seen, although there is
    a mass effect in the form of erosion of the inner table of the skull vault. What is your diagnosis?

A. Meningioma

B. Arachnoid cyst

C. Epidermoid cyst

D. Dermoid cyst

E. Porencephalic cyst

A
  1. B. Arachnoid cyst

Arachnoid cysts are CSF-containing intra arachnoid cysts with ventricular communication or brain maldevelopment. They can be congenital or acquired (leptomeningeal cyst). Most are asymptomatic, but they can present with weakness, mass effect, seizure, headache, developmental delay or craniomegaly. The most common site is the middle cranial fossa anteriorly. On MRI, they are CSF density, well defined, low on T1-weighted and high on T2-weighted images, without enhancement or calcification. They can erode the inner table of the calvarium.

96
Q
  1. A 5-year-old male child is found to have ataxic gait and multiple cranial nerve palsies.
    A CT scan of the brain shows an infratentorial hypodense mass with ring enhancement indenting the fourth ventricle posteriorly. Which of the following is the most likely diagnosis?

A. Lymphoma

B. Vascular malformation

C. Metastasis

D. Brainstem glioma

E. Haemangioma

A
  1. D. Brain,stem glioma

Ring enhancement is seen in the most aggressive necrotic/cystic tumours. Displacement of the fourth ventricle is typical of brainstem glioma. Most diffuse brainstem gliomas do not enhance and do not show restricted diffusion.
Classic medulloblastoma typically arises from the roof of the fourth ventricle and is midline in location in 75% 90% of cases. They are hyperdense on CT; show contrast enhancement in >90% and show restricted diffusion. Atypical teratoid-rhabdoid tumour mimics medulloblastomas radiologically and histologically and have been often misdiagnosed in the past.

97
Q
  1. All of the following are recognised indications for transjugular intrahepatic portosystemic shunt (TIPS), except

A. Right heart failure

B. Budd-Chiari

C. Refractory ascites

D. Acute variceal bleeding

E. Portal hypertensive gastropathy

A
  1. A. Right heart failure

The TIPS procedure is effective in achieving portal decompression and in managing some of the major complications of portal hypertension.
Indications for TIPS include variceal bleeding, secondary prevention, acute bleeding refractory to medical and endoscopic treatments, refractory ascites, hepatorenal syndrome, Budd-Chiari syndrome, hepatic veno-occlusive disease, hepatic hydrothorax and portal hypertensive gastropathy

98
Q
  1. A 55-year old woman with rectal cancer diagnosed on colonoscopy undergoes a staging MRI. The scan reveals direct extension of the tumour into the vagina and spreading to five pelvic lymph nodes. No distant metastasis were identified on the staging chest and abdomen CT performed at the same visit. The TNM stage of her rectal cancer is

A. Tl N2 MO

B. T2N0M0

C. T3 N2 MO

D. T3 N1 MO

E. T4 N2 MO

A
  1. E. T4N2M0
99
Q
  1. A 57-year-old man with known staghorn calculus was brought to the A&E department acutely unwell. Urgent CT abdomen out-of-hours showed enlarged heterogeneous right kidney with fatty masses replacing the parenchyma. Extensive fat stranding was also seen in the peri- and pararenal spaces. The most likely diagnosis is

A. Xanthogranulomatous pyelonephritis

B. Emphysematous pyelonephritis

C. Ruptured kidney

D. Acute non-specific pyelonephritis

E. Acute papillary necrosis

A
  1. A. Xanthogranulomatous pyelonephritis

Xanthogranulomatous pyelonephritis (XGP) is a chronic suppurative infection in chronic renal obstruction, that is, associated with staghorn calculus. MDCT often shows an enlarged kidney, associated staghorn calculus (75%), perirenal inflammatory stranding or fluid and, low attenuation fatty masses replacing renal parenchyma. US shows hypoechoic dilated calyces with echogenic rim or with low-level internal echoes replacing the normal renal parenchyma

100
Q
  1. A 9-year-old girl was taken to her family doctor with fever and painful knee and wrists. The GP noticed a skin rash, hepatosplenomegaly and lymphadenopathy. Plain X-ray of the knee and wrist shows expansion of bones around the knee and advanced carpometacarpal arthritis. What is the likely diagnosis?

A. Still disease

B. Haemophilia

C. Sickle-cell disease

D. Psoriasis

E. Lyme disease

A
  1. A. Still disease

Still disease is a clinical manifestation of polyarticular juvenile rheumatoid arthritis characterised by fever, rash, hepatosplenomegaly and pericarditis. There is periosteal reaction of the hand phalanges and broadening of bones with cortical thickening. The presence of advance arthropathy in the hands at such a young age along with the other clinical findings would be compatible with this condition.

Lyme disease tends to follow a monoarticular pattern with involvement of the large joints, usually the knee, with the radiological findings not as profound as that of juvenile rheumatoid arthritis.

101
Q

@# 101. A 7-year-old boy undergoes a plain CT brain for recent trauma, which reveals a cystic lesion in the cerebellum with a mural nodule laterally. No calcification is evident, but enhancement of the mural nodule is seen in post-contrast images. What is the most likely diagnosis in this patient?

A. Haemangioblastoma

B. Pilocytic astrocytoma

C. Giant cell astrocytoma

D. Pleomorphic xanthoastrocytoma

E. Lymphoma

A
  1. A. Haemangioblastoma

Haemangioblastoma (HB) is a vascular tumour of the CNS. It occurs most often in the cerebellum, where it is the most common primary neoplasm in adults. Single tumours may be sporadic, but multiple tumours are almost always associated with VHL disease. The most common MR pattern of LIB is an enhancing solid mural nodule with an adjacent non-enhancing cyst.
The cyst is typically low on T1-weighted and high on T2-weighted images, but it can have areas of high T1 signal from fat or haemorrhage. HB can also look purely cystic, solid or a mural nodule with enhancing cystic wall. HB almost never calcifies.
Pilocytic astrocytoma are cystic, with larger mural nodule, calcification, thick walls and no contrast blush to the mural nodule on angiography.

102
Q
  1. A 10-year-old child is admitted with headache, visual disturbance and short stature.
    A suprasellar mass that is hyperintense on Tl and heterogeneous on T2 is identified.
    which of the following is the most likely diagnosis?

A. Rathke cleft cyst

B. Pituitary macroadenoma

C. Pituitary microadenoma

D. Craniopharyngioma

E. Medulloblastoma

A
  1. D. Craniopharyngioma

The differential diagnosis would include Rathke cleft cyst, but that is usually intrasellar and thin-walled in appearance.

Craniopharyngioma contains calcification and is heterogeneous on MR.

Medulloblastoma is usually infratentorial and within the posterior fossa arising in the roof of the fourth ventricle, appears hyperdense on CT, and shows contrast enhancement.

Macroadenoma is typically seen in an older age group, whereas microadenoma is hypointense on Tl and non- enhancing post-gadolinium.

103
Q
  1. A 67-year-old man with several episodes of acute variceal bleeding is being investigated prior to TIPS procedure. Which one of the following is not a contraindication to TIPS?

A. Tricuspid regurgitation

B. Severe congestive cardiac failure

C. Multiple hepatic cysts

D. Severe portal hypertension

E. Unrelieved biliar}’ obstruction

A
  1. D. Severe portal hypertension

The main risk factors for developing HE include age >65 years, child score >12, prior HE, placement of a large diameter stent (>10 mm) and low PPG (<5 mm Hg).

104
Q

@# 104. Which of the following is not a limitation of CT imaging in staging of rectal cancer?

A. Poor tissue characterisation.

B. Inability to identify T4 tumour.

C. Non-visualisation of the mesorectal fascia.

D. Low sensitivity, specificity and accuracy in the detection of local nodal metastasis.

E. Sensitivity, specificity and accuracy are much lower in post radiation evaluation.

A
  1. B. Inability to identify T4 tumour.

In the setting of primary rectal cancer, MRI is used to assist in staging, in identifying patients who may benefit from preoperative chemotherapy-radiation therapy, and in surgical planning.
Currently, surgical resection with stage-appropriate neoadjuvant combined-modality therapy is the mainstay in the treatment of rectal cancer. In the past decade, the increasingly widespread adoption of total mesorectal excision (TME) has resulted in a dramatic decline in the prevalence of local recurrence from 38% to less than 10%. TME is a surgical technique that entails cn bloc resection of the primary tumour and the mesorectum by means of dissection along the mesorectal fascial plane or the circumferential resection margin (CRM). The evolution of surgical techniques and the shift to neoadjuvant chemotherapy-radiation therapy, along with the prognostic heterogeneity of stage T3 tumours, necessitate accurate preoperative staging primarily in terms of tumour (T) and nodal (N) staging, depth of tumour invasion outside the muscularis propria (early versus advanced stage T3 tumours) and the relationship of the tumour to the potential CRM. Accurate assessment of these factors allows the triage of patients to up front surgical resection or short- or long course preoperative radiation therapy or chemotherapy-radiation therapy with appropriate modification of the CRM. Recent studies have shown that high-resolution MRI is a reliable and reproducible technique with high specificity’ (92%) for predicting a negative CRM, the relationship of the tumour to the CRM and the depth of tumour invasion outside the muscularis propria.

105
Q
  1. A 73 year old woman undergoing a CT urogram shows multiple filling defects in the lower third of the ureter. All of the following are possibilities, except

A. Malakoplakia

B. Pyeloureteritis cystica

C. Non-radio-opaque calculi

D. Endometriosis

E. Retrocaval ureter

A
  1. E. Retrocaval ureter

Retrocaval ureter classically shows medial deviation at L3/4, returning to a more normal position anterior to the iliac vessels. Malakoplaki are rare plaque-like intramural lesions related to chronic UTI, affecting the bladder, ureter, collecting system and even renal parenchyma. They are not premalignant. Schistosomiasis and endometriosis typically cause multiple strictures, which can resemble multiple filling defects. Ureteritis cystica and pyeloureteritis cystica arc fairly common post-inflammatory conditions resulting in ureteric filling defects. Other causes include multiple calculi (Steinstrasse), blood clots or multiple vascular collaterals.

106
Q
  1. A 56-year-old woman with previous history of a distal radius fracture presents to her GP with progressive swelling and pain of the right hand and digits. Plain radiograph shows a healed fracture but also reveals diffuse osteopaenia involving all the small bones of the right hand with subperiosteal resorption at places. A three-phase bone scan demonstrates diffuse increased and uniform uptake involving the whole wrist. What is the likely diagnosis?

A. Soft-tissue infection

B. Osteomyelitis

C. Reflex sympathetic dystrophy

D. Vitamin D deficiency

E. Scurvy

A
  1. C. Reflex sympathetic dystrophy

Reflex sympathetic dystrophy, a syndrome that causes significant discomfort, is thought to be due to ‘sympathetic overflow’, which may explain the pain, warmth and swelling of the involved extremity. At bone scintigraphy, reflex sympathetic dystrophy usually manifests as diffuse, uniformly increased uptake throughout the affected region. Occasionally, reflex sympathetic dystrophy may manifest as a focal abnormality limited to, for example, the hand or knee. Decreased radiotracer accumulation has also been described, especially in children.
Three-phase bone scanning has an accuracy of over 90% and is the radionuclide procedure of choice for diagnosing osteomyelitis in bone not affected by underlying conditions. The first (dynamic) phase reflects the relative amount of blood flow to the area of interest, whereas the second (blood pool) phase reflects the amount of activity that has extravasated into the tissues around the area of interest. The third (delayed [bone]) phase reflects the rate of bone turnover. The classic appearance of osteomyelitis on three-phase bone scans consists of focal hyperperfusion, focal hyperaemia and focally increased bone uptake. Abnormalities at radionuclide bone imaging reflect increased bone mineral turnover in general, not infection specifically. Therefore, conditions associated with increased bone mineral turnover (e.g., tumours, fractures, joint neuropathy) may mimic osteomyelitis at three-phase bone scintigraphy. Under these circumstances, three-phase bone imaging is less useful, primarily because of diminished specificity

107
Q
  1. A 56-year-old man undergoes a CT brain for recent trauma, revealing a 2 x 2 cm well-defined hyperintense focus in the right frontal lobe without any mass effect or surrounding oedema. MR shows a well-defined, mulberry-shaped area of mixed signal intensity with
    a hypointense rim on T2W images. What is the diagnosis?

A. Capillary telangiectasia

B. Cavernous angioma

C. AVM

D. Diffuse axonal injury

E. Evolving haematoma

A
  1. B. Cavernous angioma

A cavernous angioma is a ‘mulberry-like’ vascular malformation. MR! is the most sensitive modality for the diagnosis. With T2-weighted sequences, the lesion is typically characterised by an area of mixed signal intensity, with a central reticulated core and a peripheral rim of decreased signal intensity related to deposition of hemosiderin.
Diffuse axonal injury shows multifocal microbleeds as low signal on GRE or SWI sequences. AVM are identified by serpiginous filling defects with large draining veins and feeding arteries. Haem atom as will have mixed signal and will have surrounding oedema and mass effect, unless they are very small.

108
Q
  1. Which one of the following is the most common intracranial tumour in children?

A. Metastasis

B. Astrocytoma

C. Haemangioblastoma

D. Craniopharyngioma

E. None of the above

A
  1. B. Astrocytoma

About 50%-70% of all primary intracranial tumours are astrocytoma; it is also the most common paediatric brain neoplasm, accounting for 40%-50%.

109
Q

@# 109. A 56-year-old woman post-renal transplant is being evaluated by renal ultrasound.
A small amount of fluid is noted around the graft. All of the following facts are true regarding renal and vascular complications post-transplant, except

A. Loss of corticomedullary differentiation is a feature of reduced renal function.

B. Reversal of diastolic flow is specific for renal vein thrombosis.

C. Extrarenal arteriovenous fistulas are related to surgical technique.

D. Intrarenal arteriovenous fistulas are the result of percutaneous biopsy.

E. Crescentic peritransplant fluid collections immediately post-transplantation are not usually significant

A
  1. B. Reversal of diastolic flow is specific for renal vein thrombosis.

Acute tubular necrosis is the most common cause of ‘delayed graft function’, whereas chronic rejection is the most common cause of late graft loss.
US findings of diminished renal function include renal enlargement, increased cortical thickness, increased or decreased echogenicity of the renal cortex, loss of corticomedullary differentiation, prominent pyramids, collecting system thickening and effacement of the central sinus echo complex. Elevated resistive index (>0.8) is a non-specific parameter of renal transplant dysfunction; reversal of diastolic flow may also be seen. Renal artery stenosis is the most common vascular complication of transplantation. Doppler criteria include (1) velocities of greater than 2 m/sec, (2) a velocity gradient between stenotic and prestenotic segments of more than 2:1 and (3) marked distal turbulence.

Although reversal of diastolic flow is non-specific, the combination with absent venous flow is virtually diagnostic of renal vein thrombosis. Renal artery thrombosis is diagnosed by lack of intrarenal venous and arterial flow. Renal vein stenosis results in three- to fourfold increase in flow velocity. Intrarenal arteriovenous fistulas and pseudoaneurysms are the result of vascular trauma during percutaneous biopsy. Extrarenal arteriovenous fistulas and pseudoaneurysms are extremely uncommon. They typically occur as a result of surgical technique rather than percutaneous biopsy.

Small crescentic peritransplant fluid collections seen immediately after transplantation are most likely haematomas or seromas and are not significant More complex collections identified later in the post operative period with clinical evidence of infection may represent abscesses. Although the appearance of urinomas is non-specific, internal septations are seen less often than in haematomas. Lymphocele results in a rounded collection along the mid-ureter, associated with hydronephrosis.

110
Q
  1. A 76-year-old man with mid-rectal tumour rectal cancer who has had neoadjuvant chemoradiotherapy has been referred for a pelvic MRI to assess local treatment response. Which of the following findings on rectal MRI are predictors of disease recurrence post-treatment for rectal cancer?

A. Mesorectal facial involvement

B. Peritoneal reflection involvement

C. Spiculated tumour nodules in the perirectal fat

D. Tumour involvement of pelvic wall lymph nodes

E. All of the above

A
  1. E. All of the above

High-spatial resolution MRI has already been established as an accurate tool for the preoperative staging of rectal cancer and has resulted in marked improvements in staging accuracy compared with historic studies.
MRI also defines the relationship between a tumour and the mesorectal fascia, which denotes the CRM at TME. The potential CRM is considered involved if the tumour extends to within 1 mm of this fascia.
Patients with locally advanced T3 or T4 disease or disease involving the potential CRM on baseline MRI are offered chemoradiation therapy (CRT). This approach has been shown to decrease the post-operative tumour recurrence rate.
The Magnetic Resonance Imaging and Rectal Cancer European Equivalence (MERCURY) study evaluated consecutive patients undergoing both primary surgery and preoperative therapy with histopathologic correlation and analysed survival outcomes.
The results of the study showed that post-CRT MRI assessment of tumour regression grade correlated with disease free survival and overall survival and, thus, with patient prognosis. Furthermore, post treatment MRI prediction of potential CRM involvement also gave prognostic information regarding the risk of local recurrence.

111
Q
  1. An 82-year-old man complaining of increased urgency of micturition shows a markedly reduced bladder volume on ultrasound. All the following are possible explanations, except
    A. TB cystitis
    B. Schistosomiasis
    C. Previous surgery
    D. Enlarged prostate
    E. History of pelvic radiation
A
  1. D. Enlarged prostate

Causes of reduced bladder volume include interstitial cystitis, TB cystitis (thimble bladder), cystitis cystica, schistosomiasis, surgical resection and radiation therapy. Prostate enlargement, urethral strictures and Marion’s disease (primary bladder neck obstruction) result in increased bladder volume.

112
Q

@# 112. A wrist arthrogram is performed for instability and 5 mL of contrast is injected into the radiocarpal joint. The contrast is seen to extend lateral to the lunate into the mid carpal joint Which of the following is this diagnostic of?

A. Contrast extravasation secondary to the large volume of contrast injected

B. Scapholunate ligament tear

C. Lunatotriquetral ligament tear

D. Distal radioulnar joint (DRUJ) disruption

E. Triangular fibrocartilage (TFC) tear

A
  1. B. Scapholunate ligament tear

Single-compartment and three-compartment injection techniques have been described for wrist magnetic resonance arthrography.

In the three compartment technique, the mid-carpal and distal radio ulnar joints are initially injected with approximately 1-1.5 ml. of iodinated contrast under fluoroscopic guidance. About 3 ml. of dilute gadolinium is then injected into the radiocarpal compartment after confirmation of the needle-tip position with the injection of a small volume of iodinated contrast.

Contrast should normally be confined to the radiocarpal joint with no leak of contrast into the DRUJ proximally or the mid carpal joint distally.

If contrast extends lateral to the lunate, between it and the scaphoid this is diagnostic of a full thickness scapholunate ligament tear.

113
Q
  1. A 52-year-old obtunded patient is admitted to ICU with severely deranged blood electrolytes, in particular very low sodium, and has been on intravenous fluids for 24 hours. MR brain performed to find a cause for developing spastic quadriparesis shows a central area of low Tl, high T2 signal in the pons. What is your diagnosis?

A. Pontine infarct

B. Osmotic myelinolysis

C. MS

D. Pontine haemorrhage

E. Pontine glioma

A
  1. B. Osmotic myelinolysis

Patients with osmotic demyelination syndrome typically present with severe electrolyte disturbances, which lead to seizures or encephalopathy.
A symmetric trident-shaped area in the central pons is a characteristic finding on T2-weighted and FLAIR MRI. The ventrolateral pons and the pontine portion of the corticospinal tracts arc typically spared. Decreased signal intensity, with no mass effect, is a classic finding on ‘Ll -weighted images. Less commonly, lesions appear isointense relative to surrounding brain tissue on Tl and do not enhance after the administration of contrast material. Case reports have suggested that restricted diffusion may be seen earlier than the classic T2 changes.

It has been noted that myelinolysis can occur outside the pons, a condition that is referred to as extrapontine myelinolysis.

Sites of extrapontine myelinolysis include the basal ganglia and cerebral white matter and, less commonly, the peripheral cortex, hippocampi and lateral geniculate bodies. Extrapontine myelinolysis occurs in conjunction with central pontine myelinolysis; however, it may also be seen in isolation.

114
Q

@# 114. A child is admitted to the A&E department following head injury. A CT scan of the head is performed and, incidentally, a well-defined lobulated heterogeneous mass containing calcification is seen to arise from the floor of the fourth ventricle. Which of the following is the most likely diagnosis?

A. Glioma

B. Pilocytic astrocytoma

C. Medulloblastoma

D. Ependymoma

E. Brainstem glioma

A
  1. D. Ependymoma

Medulloblastoma is the most common neoplasm of the posterior fossa in childhood and is the most malignant infratentorial tumour; however, it arises from the roof of the fourth ventricle. Calcification is an atypical feature of medulloblastoma. Brainstem gliomas displace the fourth ventricle. Pilocytic astrocytoma commonly occurs in the cerebellar hemispheres, not in the fourth ventricle.
Ependymoma is the third most common posterior fossa tumour in children. The point of origin is the floor of the fourth ventricle in ependymoma, versus the roof in medulloblastoma. Calcification is a common feature seen in 50% of ependymoma cases, and contrast enhancement is heterogeneous. Although not pathognomonic, the plastic nature of ependymoma results in the classic presentation of a fourth ventricle mass extending through the foramen of Luschka (15%) or foramen of Magendie (60%).

115
Q
  1. On post-transplant renal graft, the ultrasound resistive index can be increased by all, except

A. Infection

B. Acute rejection

C. Acute tubular necrosis

D. Renal artery stenosis

E. Extrinsic compression

A
  1. A. Infection

There is a general agreement that 0.70 should be considered the upper limit of normality in adults but not in children, in whom resistive index values are typically higher, especially within the first year of life.
Renal resistive index (RRI) is used for assessment of chronic renal allograft rejection, detection and management of renal artery stenosis, evaluation of progression of risk in chronic kidney disease, differential diagnosis in acute and chronic obstructive renal disease, and more recently as a predictor of renal and overall outcome in the critically ill patient.

116
Q
  1. A 31-year-old man with known history of Crohn’s disease needs imaging for assessment of his disease status. The gastroenterologist had requested MR enterography, but the patient is reluctant to have this due to claustrophobia. A CT enterography has been booked instead. All of the following statements regarding CT enterography for Crohn’s disease are true, except

A. The CT features of active Crohn’s disease include mucosal hyperenhancement, wall thickening (thickness >3 mm), mural stratification with a prominent vasa recta (comb sign) and mesenteric fat stranding.

B. Mural enhancement is the most sensitive indicator of active Crohn’s disease.

C. Prominence of the vasa recta adjacent to the inflamed loop of bowel (comb sign), along with increased mesenteric fat attenuation, is the most specific CT feature of active Crohn’s disease.

D. Findings that might be seen in active long-standing Crohn’s disease include submucosal fat deposition, pseudosacculation, surrounding fibro-fatty proliferation and fibrotic strictures.

E. CT enterography has high sensitivity for the detection of bowel strictures occurring as a complication of Crohn’s disease.

A
  1. D. Findings that might be seen in active long-standing Crohn’s disease include submucosal fat deposition, pseudosacculation, surrounding fibro-fatty proliferation and fibrotic strictures.

The main diagnostic purpose of CT enterography in the setting of Crohn’s disease is to differentiate active inflammatory strictures from fibrotic strictures in order to guide therapy.
CT features of active Crohn’s disease include mucosal hyperenhancement, wall thickening (thickness >3 mm), mural stratification with a prominent vasa recta (comb sign) and mesenteric fat stranding, all of which are exquisitely demonstrated at CT enterography. The capability of CT enterography for depicting extra-enteric disease allows the simultaneous diagnosis of complications associated with Crohn’s disease, such as obstruction, sinus tract, fistula and abscess formation.
Mural enhancement is the most sensitive indicator of active Crohn’s disease. Care should be taken to compare bowel loops with similar distention, since both the jejunum and normal collapsed loops may demonstrate regions of higher attenuation simulating enhancement. Inadequately distended bowel loops may be difficult to assess, and secondary signs of active disease, such as mesenteric fat stranding, vasa recta prominence or complications such as fistulas and abscesses should be sought to maximise the accuracy of a diagnosis of active disease.
Prominence of the vasa recta adjacent to the inflamed loop of bowel (comb sign), along with increased mesenteric fat attenuation, is the most specific CT feature of active Crohn’s disease. Findings that might be seen in inactive long-standing Crohn’s disease include submucosal fat deposition, pseudosacculation, surrounding fibro-fatty proliferation and fibrotic strictures.

117
Q
  1. MR angiogram performed on a patient with fibromuscular dysplasia shows a small renal artery aneurysm. Which of the following statements regarding rupture of renal artery aneurysm is incorrect?

A. Size greater than 2 cm increases the risk of rupture.

B. There is debate over whether calcification reduces risk of rupture.

C. Risk of death is high in renal artery aneurysm rupture in non-pregnant patients.

D. Pregnancy is associated with increased risk of rupture.

E. Kidney salvage may be possible in some patients.

A
  1. C. Risk of death is high in renal artery aneurysm rupture in non-pregnant patients.

The decision to repair a renal artery aneurysm (RAA) should be based on several factors,
including patient age, gender, anticipated pregnancy in female patients and anatomic features including its size. An isolated aneurysm smaller than 2 cm is unlikely to rupture.
Rupture of RAAs is unlikely in most patients. Although some consider calcification to be protective of rupture, no correlation between calcification and risk of rupture is evident. Rupture has historically been associated with a high death rate, especially during pregnancy, and pregnancy is associated with an increased risk of aneurysm rupture. In non-pregnant patients, RAA rupture is likely to be associated with death in less than 10% of cases. Reports suggest that nephrectomy is not always a certain outcome of a RAA rupture, and attempts at kidney salvage in properly selected patients can be justified.

118
Q
  1. An MRI knee of a 65-year-old woman with known osteoarthritis shows a 6 x 5 cm well-defined cystic area, posteromedial to the tibiofemoral joint on axial images. A narrow neck is identified extending between the medial head of the gastrocnemius and semimembranosus tendon to communicate with the knee joint. No significant joint effusion is evident. What is the likely diagnosis?

A. Pes anserinus bursitis

B. Baker’s cyst

C. Large parameniscal cyst

D. Tibiofibular cyst

E. Cruciate cyst

A
  1. B. Baker’s cyst

A Baker’s cyst, or popliteal cyst, is essentially a communicating synovial cyst from the posterior joint capsule. The location of the cyst, which tracks between the medial head of the gastrocnemius and semimembranosus tendon, is characteristic.

Pes anserinus bursitis refers to inflammation of the bursa around the conjoined tendon insertions of the gracilis, sartorius and semitendinosus muscles at the anteromedial aspect of the proximal tibia.

A large parameniscal cyst is a differential, but this requires a meniscal horizontal tear and no mention of this is made.

119
Q
  1. All of the following are associations of Chiari I malformation, except

A. Basilar impression

B. Klippel Feil anomaly

C. Hydrocephalous

D. Myelomeningocele

E. Syringomyelia

A
  1. D. Myelomeningocele

Chiari I malformation, also called cerebellar tonsillar ectopia, is often an abnormality affecting the hind brain in isolation. Associations of Chiari I malformation include syringohydromyelia, hydrocephalus, basilar impression or basilar invagination, occipitalization of the atlas, unfused posterior arch of Cl, platybasia and Klippel-Feil anomaly.

Myelomeningocele are not associated with Chiari 1 but Chiari II malformation.

120
Q
  1. A 7-year old child is noticed to have difficulty walking. A central mass is suspected,
    and MRI demonstrates a large cystic mass in the right cerebellar hemisphere, with an intensely enhancing nodule on post-gadolinium images. Which of the following is the most likely diagnosis?

A. Haemangioblastoma

B. Cerebellar pilocytic astrocytoma

C. Medulloblastoma

D. Ependymoma

E. Arachnoid cyst

A
  1. B. Cerebellar pilocytic astrocytoma

The classic imaging appearance of a JPA, which is observed in 30%-60% of cases, is of a large cyst with a solid mural nodule within one of the cerebellar hemispheres; less commonly, JPA may present on imaging as a predominantly solid mass with little to no cyst-1 ike component. Enhancement patterns may vary, but JPA most commonly (46%) appears as a cyst with an enhancing wall and an intensely enhancing mural nodule. DWT of )PAs shows no restricted diffusion, which is consistent with the characteristics of a low grade tumour.
Although both haemangioblastoma and pilocytic astrocytoma appear similar, the latter is more common within the first two decades, with peak between birth and 9 years of age. Medulloblastoma and ependymoma occur in the fourth ventricle. Arachnoid cyst does not enhance post-contrast.